monthly news diary september 2021

92

Upload: others

Post on 30-Jan-2022

7 views

Category:

Documents


0 download

TRANSCRIPT

Page 1: MONTHLY NEWS DIARY SEPTEMBER 2021
Page 2: MONTHLY NEWS DIARY SEPTEMBER 2021

MONTHLY NEWS DIARY SEPTEMBER 2021

www.sosinclasses.com +91 90000 36699 [email protected]

Pag

e2

Page 3: MONTHLY NEWS DIARY SEPTEMBER 2021
Page 4: MONTHLY NEWS DIARY SEPTEMBER 2021

MONTHLY NEWS DIARY SEPTEMBER 2021

www.sosinclasses.com +91 90000 36699 [email protected]

Pag

e1

Page 5: MONTHLY NEWS DIARY SEPTEMBER 2021

MONTHLY NEWS DIARY SEPTEMBER 2021

www.sosinclasses.com +91 90000 36699 [email protected]

Pag

e2

INDEX

GS – 1

1. INDIAN HISTORY

1.1. Thamirabarani Civilisation: Tamil Nadu…………………………………………………………………………………04

1.2. Gupta Period Temple Remains Found: UP…………………………………………………………………………….05

2. GEOGRAPHY

2.1. Footprints of 3 Dinosaur Species: Rajasthan………………………………………………………………………….07

3. PRELIMS WORKBOOK………………………………………………………………………………………………………08

GS – 2

1. POLITY & GOVERNANCE

1.1. ATL Space Challenge 2021……………………………………………………………….……………………………………12

1.2. Swachh Survekshan Grameen 2021………………………………………………………………………………………13

1.3. Shoonya Campaign: NITI Aayog….…………………………………………………………………………………………14

1.4. Crime in India Report 2020: NCRB…………………………………………………………………………………………16

1.5. Urban Planning Reforms: NITI Aayog.……………………………………………………………………………………18

1.6. G-20 Agriculture Meet 2021………….………………………………………………………………………………………20

1.7. Front-of-Pack Labelling………………………………………………………………………………………………………….21

1.8. Rail Kaushal Vikas Yojana…………………..………………………………………………………………………………….23

1.9. State Food Safety Index: FSSAI………………..…………………………………………………………………………….25

1.10. Forest Right Act………………………………………………………………………………………………………….26

2. INTERNATIONAL RELATIONS 2.1. Resolution 2593 on Taliban: UNSC….…………………………………………………………………………………….28

2.2. China-Myanmar New Passage……….……………………………………………………………………………………..29

2.3. India-Australia First 2+2 Dialogue…..………………………………………………………………………………..…..31

2.4. US-India Strategic Clean Energy Partnership (SCEP)………………………………………………………………33

2.5. Climate Action and Finance Mobilisation Dialogue: India-US.……………………………………………….34

2.6. Connectivity Projects: India and Southeast Asia…………………………………………………………………….35

2.7. AUKUS Grouping……………………………………………………………………………………………………………………37

2.8. 15th East Asia Summit Energy Ministers Meeting…………………………………………………………………..38

3. PRELIMS WORKBOOK……………………………………………………………………………………………………..41

Page 6: MONTHLY NEWS DIARY SEPTEMBER 2021

MONTHLY NEWS DIARY SEPTEMBER 2021

www.sosinclasses.com +91 90000 36699 [email protected]

Pag

e3

GS – 3

1. ECONOMY 1.1. Sharp Economic Recovery……………………….……………………………………………………………………………47 1.2. T+1 Settlement System for Shares: SEBI………………………………………………………………………………..49 1.3. IRDAI Guidelines 2021……………………………..……………………………………………………………………………50 1.4. No Modification Under the Resolution Plan Under IBC..……………………………………………………….51 1.5. New Bad Bank Structure………………….……………………………………………………………………………………53 1.6. PLI Scheme for Auto & Drone………………………………………………………………………………………………..54 1.7. Sufficiency Economy Philosophy: Thailand…………………………………………………………………………….56 1.8. World Bank Stops ‘Ease of Doing Business’ Report………………………………………………………………..58

2. ENVIRONMENT 2.1. Managing Natural Resources …..…………………………………………………………………………………………..60 2.2. Mumbai Climate Action Plan (MCAP)..………………………………………………………………………………….61 2.3. Creating Methanol from Coal……………………………………………………………………………………………….62 2.4. Changing Rainfall Pattern in North-East India………………………………………………………………………..64 2.5. Impact of Global Warming on Permafrost……………………………………………………………………………..65 2.6. Task Force for Coal Based Hydrogen Production……………………………………………………………………67 2.7. Arsenic Contamination of Food Chain……………………………………………………………………………………69 2.8. United in Science 2021: WMO………………………………………………………………………………………………70 2.9. World’s First ‘Five-Country Biosphere Reserve’.……………………………………………………………………72

3. SCIENCE & TECHNOLOGY 3.1. Lunar Science Workshop 2021………………………………………………………………………………………………75 3.2. Restraining Mosquito Populations with CRISPR...………………………………………………………………….76 3.3. Robots in Warfare…………………………………………………………………………………………………………………78 3.4. Mu Variant of Covid-19…………………………………………………………………………………………………………79 3.5. Role of Rice in Boosting Nourishment……………………………………………………………………………………80

4. PRELIMS WORKBOOK………………………………………………………………………………………………………82 ANSWER KEY………………………………………………………………………………………………………….………...85

Page 7: MONTHLY NEWS DIARY SEPTEMBER 2021

MONTHLY NEWS DIARY SEPTEMBER 2021

www.sosinclasses.com +91 90000 36699 [email protected]

Pag

e4

Page 8: MONTHLY NEWS DIARY SEPTEMBER 2021

MONTHLY NEWS DIARY SEPTEMBER 2021

www.sosinclasses.com +91 90000 36699 [email protected]

Pag

e5

Page 9: MONTHLY NEWS DIARY SEPTEMBER 2021

MONTHLY NEWS DIARY SEPTEMBER 2021

www.sosinclasses.com +91 90000 36699 [email protected]

Pag

e6

GS - 1

1. INDIAN HISTORY

1.1. DISCUSS THE SIGNIFICANCE OF THE FINDINGS IN TAMIL NADU ABOUT ‘THAMIRABARANI

CIVILIZATION’.

Background: The Thamirabarani civilization in Tamil Nadu is at least 3,200 years old, reveals carbon dating done on organic material retrieved from archeological excavations in Sivakalai, Thoothukudi district.

Thamirabarani River:

The shortest river in the state (Tamil Nadu), the Thamirabarani starts in Pothigai hills of the Western Ghats in the Ambasamudram taluk, flows through Tirunelveli and Thoothukudi districts and empties at Korkai (Tirunelveli district) into the Gulf of Mannar (Bay of Bengal).

Significance of the Findings:

➢ It could lead to evidence that there was a city civilisation (Porunai River (Thamirabarani) civilization) in south India as long back as 3,200 years ago, the later part of the Indus Valley Civilisation.

➢ Also, archaeological excavations would be carried out in other States and countries in search of Tamil roots.

Carbon Dating

The determination of the age or date of organic matter from the relative proportions of the carbon isotopes carbon-12 and carbon-14 that it contains.

Page 10: MONTHLY NEWS DIARY SEPTEMBER 2021

MONTHLY NEWS DIARY SEPTEMBER 2021

www.sosinclasses.com +91 90000 36699 [email protected]

Pag

e7

➢ In the first phase, studies would be undertaken at the ancient port of Muziris, now known as Pattanam, in Kerala, to establish the ancientness and culture of the Chera empire.

➢ Research would be conducted at Quseir al-Qadim and Pernica Anekke in Egypt, which were once part of the Roman empire, as well as in Khor Rori in Oman, to establish the Tamils’ trade relations with these countries. Potsherds bearing Tamil scripts have been found in these countries.

➢ Studies would also be conducted in Southeast Asian countries, such as Indonesia, Thailand, Malaysia and Vietnam, where King Rajendra Chola had established supremacy.

➢ The three ruling houses of Tamil India, the Pandyas, Cheras, and Cholas, fought for supremacy of southern India and Sri Lanka. These dynasties promoted early literature on the Indian subcontinent and built important Hindu temples.

➢ Sangam literature, which was written over a period of six centuries (3rd BCE – 3rd CE) contains references to various Chola, Chera and Pandya kings.

Other Recent Findings:

A silver punch marked coin was recently excavated from Keezhadi (Tamil Nadu) which bore the symbols of the sun, the moon, the taurine and other geometrical patterns. Studies on this found the coin to date back to the 4th century BC, which is before the time of the ancient Maurya empire (321-185 B.C.E). Archaeological excavations are being done at numerous places such as Kodumanal, Keeladi, Korkai, Sivakalai, in Tamil Nadu. As per the carbon dating of artefacts, the Keeladi civilisation dates to 6th century BC.

1.2. EXAMINE THE FINDINGS OF THE GUPTA PERIOD TEMPLE REMAINS IN UTTAR PRADESH.

Background: Recently, the Archaeological Survey of India (ASI) discovered remains of an ancient temple dating back to the Gupta period (5th century) in Bilsarh village in Uttar Pradesh’s Etah district. The Bilsarh site was declared ‘protected’ in 1928 by the ASI.

About:

Two pillars were excavated on which there is an inscription about Kumaragupta I, the powerful ruler of Gupta dynasty, in ‘sankh lipi’ (conch script or shell script) typical of the 5th century AD.

The Guptas were the first to build structural temples, distinctly different from the ancient rock-cut temples.

The inscription was deciphered as Mahendraditya which was the title of king Kumargupta I who had even conducted ashwamedh yagya during his rule.

The statue of a horse having similar inscription is at the state museum in Lucknow. Ashvamedha Yagya is a horse sacrifice ritual followed by the Srauta tradition of Vedic

religion. The discovery becomes significant since only two other structural temples from the

Gupta age have been found so far — Dashavatara Temple (Deogarh) and Bhitargaon Temple (Kanpur Dehat).

Page 11: MONTHLY NEWS DIARY SEPTEMBER 2021

MONTHLY NEWS DIARY SEPTEMBER 2021

www.sosinclasses.com +91 90000 36699 [email protected]

Pag

e8

Shankhalipi Script: Also called “shell-script” is found in inscriptions across north-central India and dates to between the 4th and 8th centuries. Both Shankhalipi and Brahmi are stylised scripts used primarily for names and signatures. The inscriptions consist of a small number of characters, suggesting that the shell inscriptions are names or auspicious symbols or a combination of the two. It was discovered in 1836 on a brass trident in Uttarakhand’s Barahat by English scholar James Prinsep. Prominent sites with shell inscriptions: Mundeshwari Temple (Bihar), the Udayagiri Caves (Madhya Pradesh), Mansar (Maharashtra) and some of the cave sites of Gujarat and Maharashtra. These inscriptions are also reported in Indonesia’s Java and Borneo.

Kumaragupta-I:

He was the Successor of Chandragupta-II and had a long reign from 414 to 455 A.D. He performed the Asvamedha Yajna which was confirmed by Asvamedha coins. The discovery of his 1395 coins confirms his extension towards the South. His period is also regarded as forming part of the Golden Age of the Guptas. Towards the middle of the fifth century A.D. The reign of the Kumaragupta-I was disturbed by the revolt of Pushyamitra tribe and the invasion of the Hunas. His greatest achievement was his repulsing the attack of the Pushyamitra. After the death of Kumaragupta-I Skandagupta succeeded in 455 A.D. and reigned from 455 to 467 A.D.

Gupta Empire

The Gupta Empire stretched across northern, central and parts of southern India between 320 and 550 CE.

The period is noted for its achievements in the arts, architecture, sciences, religion, and philosophy.

Chandragupta I (320 – 335 CE) started a rapid expansion of the Gupta Empire and soon established himself as the first sovereign ruler of the empire.

It marked the end of 500 hundred years of domination of the provincial powers and resulting disquiet that began with the fall of the Mauryas.

It began a period of overall prosperity and growth that continued for the next two and half centuries which came to be known as a Golden Age in India's history.

Page 12: MONTHLY NEWS DIARY SEPTEMBER 2021

MONTHLY NEWS DIARY SEPTEMBER 2021

www.sosinclasses.com +91 90000 36699 [email protected]

Pag

e9

2. GEOGRAPHY 2.1. “FOOTPRINTS OF 3 DINOSAUR SPECIES” FOUND IN RAJASTHAN. COMMENT.

Background: Recently, in a major discovery, footprints of three species of dinosaurs have been found in the Thar desert in Rajasthan’s Jaisalmer district. It proves the presence of the giant reptiles in the western part of the State.

About the Discovery:

➢ The footprints belong to three species of dinosaurs - Eubrontes cf. giganteus, Eubrontes

glenrosensis and Grallator tenuis.

➢ The footprints were 200 million years old.

➢ The dinosaur species are of the theropod type, with the distinguishing features of

hollow bones and feet with three digits (like fingers).

➢ Theropod is any member of the dinosaur subgroup Theropoda, which includes all the

flesh-eating dinosaurs.

➢ All the three species, belonging to the early Jurassic period, were carnivorous.

➢ The 'Age of Dinosaurs' (the Mesozoic Era - 252-66 Million Years Ago - MYA) included

three consecutive geologic time periods (the Triassic, Jurassic, and Cretaceous Periods).

Different dinosaur species lived during each of these three periods.

THAR DESERT

Naming: The name ‘Thar’ is derived from thul, the general term for the region’s sand ridges. It is also called the ‘Great Indian Desert’. Location: Partly in Rajasthan state, northwestern India, and partly in Punjab and Sindh provinces, eastern Pakistan. It is bordered by the irrigated Indus River plain to the west, the Punjab Plain to the north and northeast, the Aravalli Range to the southeast, and the Rann of Kachchh to the south.

Page 13: MONTHLY NEWS DIARY SEPTEMBER 2021

MONTHLY NEWS DIARY SEPTEMBER 2021

www.sosinclasses.com +91 90000 36699 [email protected]

Pag

e10

3. PRELIMS WORKBOOK 1. The national motto of India, ‘Satyameva Jayate’ inscribed below the Emblem of India is

taken from: a) Katha Upanishad b) Chandogya Upanishad c) Aitareya Upanishad d) Mundaka Upanishad 2. Consider the following statements: 1) The Khilafat movement was launched under the leadership of the Ali Brothers. 2) Treaty of Versailles was a treaty signed between the Allies of World War I and the Ottoman

Empire. Which of the above statements is/are correct?

a) 1 only b) 2 only c) Both d) None 3. Consider the following statements: 1) He founded the Swadeshi Steam Navigation Company (SSNC) in 1906 to compete against

the monopoly of the British India Steam Navigation Company. 2) He was a great believer in the Swadeshi movement. 3) He is sometimes called ‘Kappalottiya Tamilan’.

The above statements best describe: a) Subramanya Bharathi b) Chidambaram Pillai c) Rajagopalachari d) E.V.Ramasamy 4. Therukoothu, a street theatre form, is practised in the State of: a) Andhra Pradesh b) Karnataka c) Kerala d) Tamil Nadu 5. Consider the following statements with respect to Jahanpanah City: 1) It was the last medieval city of Delhi. 2) It was built by Sultan Firuz Shah Tughlaq.

Which of the above statements is/are incorrect? a) 1 only b) 2 only c) Both d) None 6. With respect to Rabindranath Tagore, which of the following statements is/are correct?

1) Tagore renounced his Knighthood in protest of the Bengal famine. 2) He became the first non-European to win the Nobel Prize for Literature. 3) The national anthem of Bangladesh was composed by Tagore.

Options: a) 1 and 2 only b) 2 and 3 only c) 1 and 3 only d) 1, 2 and 3

7. Judima Rice Wine’, a popular beverage in the North-East, is brewed by which indigenous

tribe? a) Bodos of Assam b) Kukis of Mizoram c) Dimasa tribe of Assam d) Garo tribe of Meghalaya 8. Consider the following statements: 1) This river rises near Sihawa in the Raipur district of Chhattisgarh. 2) It is one of the most active silt-depositing streams in the Indian subcontinent.

Page 14: MONTHLY NEWS DIARY SEPTEMBER 2021

MONTHLY NEWS DIARY SEPTEMBER 2021

www.sosinclasses.com +91 90000 36699 [email protected]

Pag

e11

3) Hirakud Dam is built across this river. The above statements best describe:

a) Bedti b) Subarnarekha c) Mahanadi d) Shetrunji 9. Which of the following statements with respect to the Swadeshi Movement is/are

incorrect? 1) It is also known as the Vande Mataram Movement. 2) The movement spread to Madras under Chidambaram Pillai. 3) It remained confined to the upper and middle classes and zamindars and failed to reach

masses especially the peasantry. Options:

a) 2 and 3 only b) 1 only c) 3 only d) None of the above 10. Which of the following are the Arya Satyas in Buddhism? 1) Dukkha 2) Samudaya 3) Sangha 4) Nirodha 5) Magga

Options:

a) 1, 2, 3 and 4 only b) 2, 3, 4 and 5 only c) 1, 2, 4 and 5 only d) 1, 2, 3, 4 and 5

Page 15: MONTHLY NEWS DIARY SEPTEMBER 2021

MONTHLY NEWS DIARY SEPTEMBER 2021

www.sosinclasses.com +91 90000 36699 [email protected]

Pag

e12

Page 16: MONTHLY NEWS DIARY SEPTEMBER 2021

MONTHLY NEWS DIARY SEPTEMBER 2021

www.sosinclasses.com +91 90000 36699 [email protected]

Pag

e13

Page 17: MONTHLY NEWS DIARY SEPTEMBER 2021

MONTHLY NEWS DIARY SEPTEMBER 2021

www.sosinclasses.com +91 90000 36699 [email protected]

Pag

e14

GS - 2

1. POLITY & GOVERNANCE

1.1. EXAMINE ‘ATL SPACE CHALLENGE 2021’ AND INITIATIVES TAKEN UNDER IT.

Background: Recently, NITI Aayog’s Atal Innovation Mission (AIM) in collaboration with ISRO (Indian Space Research Organisation) and CBSE (Central Board of Secondary Education) has launched 'ATL Space Challenge 2021’. Earlier, the Union cabinet in June 2020 decided to open up the space sector for private sector participation, and cleared the creation of a new entity, Indian National Space Promotion and Authorization Centre (IN-SPACe), to encourage private companies to use India’s space infrastructure.

Atal Innovation Mission:

▪ AIM is Government of India’s flagship initiative to promote a culture of innovation and entrepreneurship in the country.

▪ Its objective is to develop new programmes and policies for fostering innovation in different sectors of the economy, provide platform and collaboration opportunities for different stakeholders, create awareness and create an umbrella structure to oversee the innovation ecosystem of the country.

Major Initiatives:

➢ Atal Tinkering Labs: Creating problem solving mindset across schools in India.

➢ Atal Incubation Centers: Fostering world class startups and adding a new dimension to the incubator model.

➢ Atal New India Challenges: Fostering product innovations and aligning them to the needs of various sectors/ministries.

➢ Mentor India Campaign: A national mentor network in collaboration with

ABOUT This is to ensure that students of classes 6 to 12 are

given an open platform where they can innovate and enable themselves to solve digital age space technology problems.

Students from both ATL (Atal Tinkering Labs) and non-ATL schools can submit their entries in the space challenge. School teachers, ATL In-charges and mentors may support the student teams.

Under the ATL initiative, schools receive a grant for setting up such labs to encourage students to give shape to their ideas and come up with innovative experiments on their own in these laboratories.

It aligns with the World Space Week 2021 which is observed from 4th -10th October each year, to celebrate the contributions of space science and technology.

This comes against the backdrop of India’s push to move up in the Global Innovation Index (GII, released by the World Intellectual Property Organisation) rankings.

Page 18: MONTHLY NEWS DIARY SEPTEMBER 2021

MONTHLY NEWS DIARY SEPTEMBER 2021

www.sosinclasses.com +91 90000 36699 [email protected]

Pag

e15

the public sector, corporates, and institutions, to support all the initiatives of the mission.

➢ Atal Community Innovation Center: To stimulate community centric innovation and ideas in the unserved /underserved regions of the country including Tier 2 and Tier 3 cities.

❖ Atal Research and Innovation for Small Enterprises (ARISE): To stimulate innovation and research in the MSME industry.

1.2. ASSESS THE FEATURES OF SWACHH SURVEKSHAN GRAMEEN 2021 SCHEME.

Background: Recently, the Jal Shakti Ministry launched Swachh Survekshan Grameen 2021 or rural cleanliness survey 2021 under Swachh Bharat Mission (Grameen) Phase-II. Before this, the Swachh Survekshan Grameen had been carried out by the Ministry in 2018 and 2019.The Swachh Survekshan Urban 2021 is to be announced. It was introduced by the Ministry of Housing and Urban Affairs (MoHUA) in 2016.

Swachh Survekshan Grameen 2021:

➢ Assesses the state of hygiene, cleanliness and sanitation in rural India as a part of the Centre’s initiative to award Open Defecation Free (ODF) Plus status to villages.

➢ ODF-plus status aims to ensure management of solid as well as liquid waste and is an upgradation of ODF status in which construction of adequate toilets was required so that people did not have to defecate in the open. It is carried out by an expert agency.

➢ Coverage: As part of the 2021 rural cleanliness survey, as many as 17,475 villages spanning across 698 districts would be covered.

Swachh Bharat Mission (Grameen) Phase-II:

➢ It emphasizes the sustainability of achievements under phase I and to provide adequate facilities for Solid/Liquid & plastic Waste Management (SLWM) in rural India.

➢ Implementation: It will be implemented from 2020-21 to 2024-25 in a mission mode with a total outlay of Rs. 1,40,881 crores.

➢ Funding Pattern: The fund sharing pattern between Centre and States will be 90:10 for North-Eastern States and Himalayan States and UT of J&K; 60:40 for other States; and 100% for other Union Territories.

➢ Funding Norms for SLWM have been rationalized and changed to per capita basis in place of the number of households.

Page 19: MONTHLY NEWS DIARY SEPTEMBER 2021

MONTHLY NEWS DIARY SEPTEMBER 2021

www.sosinclasses.com +91 90000 36699 [email protected]

Pag

e16

Swachh Bharat Mission To accelerate the efforts to achieve universal sanitation coverage, the Prime Minister of

India had launched the Swachh Bharat Mission on 2nd October 2014. Under the mission, all villages, Gram Panchayats, Districts, States and Union Territories

in India declared themselves "Open-Defecation Free" (ODF) by 2nd October 2019, the 150th birth anniversary of Mahatma Gandhi, by constructing over 100 million toilets in rural India.

SBM is being implemented by the Ministry of Housing and Urban Affairs (M/o HUA) and by the Ministry of Jal Shakti for urban and rural areas respectively.

In the Union Budget 2021-22, the Swachh Bharat Mission (Urban) 2.0 was announced to be implemented over five years – 2021 till 2026 – with an outlay of Rs. 1.41 lakh crore.

Swachh Bharat Mission (Grameen) Phase-II was launched in 2020. It further led to Jal Jeevan Mission (JJM) for addressing water security.

1.3. WHAT IS SHOONYA CAMPAIGN?

Background: Recently, NITI Aayog and Rocky Mountain Institute (RMI) and RMI India’s have launched the Shoonya Campaign. It is an initiative to promote zero-pollution delivery vehicles by working with consumers and industry. RMI is an independent nonprofit organization founded in 1982.

Need of EVs for Final-Mile Deliveries:

➢ Growing E-Commerce Market: Between 2013 and 2017, India’s online retail market grew at an average rate of 53% each year and is expected to become a $150 billion market by 2022. By shifting the end-transportation of goods from the consumer, this has dramatically expanded the fleets of delivery vehicles.

➢ Reduce Emissions: Urban freight vehicles account for 10% of freight transportation-related CO2 emissions in India, and these emissions are expected to grow by 114% by 2030. EVs emit no tailpipe emissions, which can contribute immensely to an improved air quality. Even when accounting for their manufacture, they emit 15-40% less CO2

About Shoonya Campaign Electric Deliveries: The campaign aims to accelerate adoption of Electric

Vehicles (EVs) in the urban deliveries segment and create consumer awareness about the benefits of zero-pollution delivery.

Shoonya Brand: A corporate branding and certification programme is being launched to recognize and promote industry’s efforts towards transitioning to EVs for final-mile deliveries. It will help e-commerce companies to distinguish their offerings from those of their competitors.

Online Tracking Platform: An online tracking platform will share the campaign’s impact through data such as vehicle kilometers electrified, carbon savings, criteria pollutant savings and other benefits from clean delivery vehicles.

Page 20: MONTHLY NEWS DIARY SEPTEMBER 2021

MONTHLY NEWS DIARY SEPTEMBER 2021

www.sosinclasses.com +91 90000 36699 [email protected]

Pag

e17

compared to their internal combustion engine counterparts and have lower operational cost.

➢ Energy Security: Shifting towards EVs will help India to reduce oil dependency while solving the challenge of energy scarcity and moving towards renewable and clean sources of energy.

Challenges:

➢ Technological: India is technologically deficient in the production of electronics that form the backbone of the EV industry, such as batteries, semiconductors, controllers, etc.

➢ Infrastructural Support: The lack of clarity over AC versus DC charging stations, grid stability and range anxiety (fear that batteries will soon run out of power) are other factors that hinder the growth of the EV industry. Availability of Materials for Domestic Production: Battery is the single most important component of EVs. India does not have any known reserves of lithium and cobalt which are required for battery production. India is dependent on countries like Japan and China for the import of lithium-ion batteries.

➢ Lack of Skilled Workers: EVs have higher servicing costs and higher levels of skills is needed for servicing. India lacks dedicated training courses for such skill development.

Initiatives Taken:

National Electric Mobility Mission Plan (NEMMP): NEMMP was launched in 2013 with an aim to achieve national fuel security by promoting hybrid and electric vehicles in the country.

FAME Scheme: The Indian government has created momentum through its Faster Adoption and Manufacturing of (Hybrid &) Electric Vehicles schemes that encourage, and in some segments mandates the adoption of EVs, with a goal of reaching 30% EV penetration by 2030.

National Mission on Transformative Mobility and Battery Storage: The Mission will recommend and drive the strategies for transformative mobility and phased manufacturing programmes for EVs, EV Components and Batteries.

Fiscal Incentives: To spur the production and consumption of EVs and charging infrastructure - such as income tax rebates, exemption from customs duties, etc.

Page 21: MONTHLY NEWS DIARY SEPTEMBER 2021

MONTHLY NEWS DIARY SEPTEMBER 2021

www.sosinclasses.com +91 90000 36699 [email protected]

Pag

e18

1.4. ANALYSE THE CRIME IN INDIA BASED ON THE REPORT OF NATIONAL CRIME RECORDS

BUREAU 2020.

Background: Recently, National Crime Records Bureau (NCRB) released the Crime in India Report 2020. While 2020, a year marked by months of a national lockdown due to the pandemic saw fewer traditional crimes such as those committed against women and children, among others, it witnessed a big spike in civil conflicts.

Riots (Civil Conflicts):

• Communal riots registered an increase of 96% in 2020 over the previous year.Delhi Police alone registered the highest i.e. 520 cases of communal riots in the year and Uttar Pradesh (UP) did not record a single case of communal violence in 2020.

• Caste riots saw an increase of close to 50%, agrarian riots 38% and riots during ‘andolan/morcha’ increased by 33%.

Traditional Crimes:

➢ The number of cases registered for crimes against women, children and senior citizens, theft, burglary, robbery and dacoity, among others, dropped by about 2 lakhs.

➢ Murder registered a marginal increase of 1% even as offences falling under the category of “violent crimes” decreased by 0.5%.

➢ Delhi is the most unsafe city for women. More than 10,093 cases of crimes against women were registered in the capital in 2020.

Environment Related Offences: Cases under the ‘environment-related offences’ category increased by 78.1% in the country in 2020.

Cyber Crime: The rate of cybercrime (incidents per lakh population) also increased from 3.3% in 2019 to 3.7% in 2020.

Page 22: MONTHLY NEWS DIARY SEPTEMBER 2021

MONTHLY NEWS DIARY SEPTEMBER 2021

www.sosinclasses.com +91 90000 36699 [email protected]

Pag

e19

Offences Against State:

▪ The year also saw a significant drop in cases related to Offences against the State, with a drop of 27% over 2019.

▪ UP, however, was the only major state to record an increase in this category, mostly due to the large number of ‘Damage to Public Property’ cases registered by the state, many of them during the anti- CAA (Citizenship (Amendment) Act, 2019) protests.

▪ Offences against the State’ include cases related to sedition and waging war against the nation, those falling under provisions of the Unlawful Activities Prevention Act (UAPA) 1967, Official Secrets Act 1923 and Damage to Public Property Act 1954.

State Wise Data:

National Crime Records Bureau

NCRB, headquartered in New Delhi, was set-up in 1986 under the Ministry of Home Affairs to function as a repository of information on crime and criminals to assist the investigators in linking crime to the perpetrators.

It was set up based on the recommendations of the National Police Commission (1977-1981) and the MHA’s Task Force (1985).

NCRB brings out the annual comprehensive statistics of crime across the country (‘Crime in India’ report).

Having been published since 1953, the report serves as a crucial tool in understanding the law-and-order situation across the country.

The inaugural function of the 2nd CCTNS Hackathon and Cyber Challenge 2020-21 of NCRB was held in New Delhi.

Page 23: MONTHLY NEWS DIARY SEPTEMBER 2021

MONTHLY NEWS DIARY SEPTEMBER 2021

www.sosinclasses.com +91 90000 36699 [email protected]

Pag

e20

1.5. ON THE ONSET OF URBAN PLANNING REFORMS UNDER NITI AAYOG, ANALYSE THE

STATE-WISE SCENARIO OF URBANIZATION IN INDIA.

Context: Recently, NITI Aayog has launched the report titled ‘Reforms in Urban Planning Capacity in India’.

Urbanisation in India:

➢ Urbanisation Level (National): India’s population stood at 1210 million in 2011, with an urbanisation level of 31.1% (Census of India 2011). Urbanisation is an increase in the number of people living in towns and cities. The distribution of urban centres and the pace of urbanisation is not uniform across the country. Over 75% of the urban population of the country is in 10 States: Maharashtra, Uttar Pradesh, Tamil Nadu, West Bengal, Andhra Pradesh, Gujarat, Karnataka, Madhya Pradesh, Rajasthan, and Kerala.

➢ State-wise Scenario: Above National Average: Goa, Tamil Nadu, Kerala, Maharashtra, and Gujarat

have attained over 40% urbanisation. Below National Average: Bihar, Odisha, Assam, and Uttar Pradesh continue to be

at a lower level of urbanisation than the national average of 31.1%. Union Territories: NCT of Delhi, Daman and Diu, Chandigarh, and Lakshadweep,

show above 75% urbanisation.

Need for Reforming Urban Planning Capacity:

➢ Increasing Urbanisation: India’s urban population is 11% of that of the world. However, in absolute numbers, the urban population in India is more than highly urbanised countries/regions like the United States, Japan, Western Europe, and South America. During 2011-36, urban growth will be responsible for 73% of the rise in total population in India.

➢ Urbanisation is Central to India’s Economy: Urbanisation contributes nearly 60% to India’s Gross Domestic Product (GDP). However, there exists large, untapped economies of scale.

Page 24: MONTHLY NEWS DIARY SEPTEMBER 2021

MONTHLY NEWS DIARY SEPTEMBER 2021

www.sosinclasses.com +91 90000 36699 [email protected]

Pag

e21

➢ India’s National Growth Targets: Economic Growth Target: USD 5 trillion economy by 2024.

➢ Employment Target: Total workforce estimated to be 0.64 billion by 2030, of which 0.26 billion to be employed in urban areas.

➢ Infrastructure Targets: Creation of 11 large industrial corridors as part of the National Industrial Corridor Programme, several multi-modal logistic parks, etc.

➢ Environmental Protection Targets: River rejuvenation, clean air in cities, etc.National ➢ Infrastructure Pipeline (NIP): The urban sector has a significant share of 17% in the NIP.

NIP facilitates infrastructure projects in the country with a projected investment of Rs 111 lakh crore during the period 2020-25.

India’s Global Commitments:

✓ SDGs (Goal 11): Promote urban planning as one of the recommended methods for achieving sustainable development.

✓ UN-Habitat’s New Urban Agenda: It was adopted at Habitat III in 2016. It puts forth principles for the planning, construction, development, management, and improvement of urban areas. UN-Habitat (2020) mentions spatial sustainability, as a concept. It suggests that the spatial conditions of a city can enhance its power to generate social, economic and environmental value and well-being.

✓ Paris Agreement: India’s National Determined Contributions (NDCs) includes the goals to reduce the emission intensity of the country's GDP by 33 to 35% by 2030 from 2005 level.

Recommendations:

➢ Planning of Healthy Cities: Central Sector Scheme ‘500 Healthy Cities Programme’, for a period of 5 years, wherein priority cities and towns would be selected jointly by the states and local bodies. The Programme can lead to optimum utilization of urban land as well.

➢ Re-engineering of Urban Governance: To bring in more institutional clarity and also multi-disciplinary expertise to solve urban challenges. The formation of an apex committee at the state level is recommended to undertake a regular review of planning legislations (including town and country planning or urban and regional development acts or other relevant acts).

➢ Strengthening the Role of the Private Sector: These include the adoption of fair processes for procuring technical consultancy services, strengthening project structuring and management skills in the public sector, and empanelment of private sector consultancies.

➢ Measures for Strengthening Human Resource and Match Demand–Supply: The constitution of a ‘National Council of Town and Country Planners’ as a statutory body of the Government of India. Also, a ‘National Digital Platform of Town and Country Planners’ is suggested to be created within the National Urban Innovation Stack of MoHUA. Citizen Outreach Campaign’ for demystifying urban planning. Strengthening Urban Planning Education System.

Page 25: MONTHLY NEWS DIARY SEPTEMBER 2021

MONTHLY NEWS DIARY SEPTEMBER 2021

www.sosinclasses.com +91 90000 36699 [email protected]

Pag

e22

1.6. EXAMINE THE SIGNIFICANCE OF THE G-20 AGRICULTURE MEET OF 2021.

Context: Recently, India’s Agriculture Minister addressed the G20 Agriculture meeting virtually. It is one of the ministerial meetings organised as part of the G20 Leaders’ Summit 2021 to be hosted by Italy in October 2021.

Highlights of the Meet:

Signed a final statement named the “Florence Sustainability Charter”. It will strengthen cooperation between G20 members and developing countries on food

and agriculture to share knowledge and help developing the internal production capacities best suited to local needs, thus contributing to the resilience and recovery of agriculture and rural communities.

Reaffirmed their intention to reach the goal of zero hunger, which is also threatened by the consequences of Covid-19.

Reaffirmed the commitment to achieve food security in the framework of the three dimensions of sustainability: economic, social and environmental.

G20

It is an informal group of 19 countries and the European Union (EU), with representatives of the International Monetary Fund and the World Bank.

It does not have a permanent secretariat or Headquarters. The membership comprises a mix of the world’s largest advanced and emerging

economies, representing about two-thirds of the world’s population, 85% of global gross domestic product, 80% of global investment and over 75% of global trade.

Members: Argentina, Australia, Brazil, Canada, China, France, Germany, India, Indonesia, Italy, Japan, Republic of Korea, Mexico, Russia, Saudi Arabia, South Africa, Turkey, the United Kingdom, the United States and the EU.

Page 26: MONTHLY NEWS DIARY SEPTEMBER 2021

MONTHLY NEWS DIARY SEPTEMBER 2021

www.sosinclasses.com +91 90000 36699 [email protected]

Pag

e23

India’s Stand:

➢ Focus on Traditional Food: Emphasised on re-introducing traditional food items including millet, other nutritious cereals, fruits and vegetables, fish, dairy and organic products in the diet of the people. Their production has been phenomenal in India in recent years and India is becoming a destination country for healthy food items. The United Nations (UN) has accepted India’s proposal and declared 2023 as the International Year of Millets and urged the G20 nations to support the celebration of the millet year to promote nutrition and sustainable agriculture.

➢ Biofortified Food: Biofortified varieties, which are the source of a staple diet rich in micronutrients, are being promoted to remove malnutrition. About 17 such varieties of different crops have been developed and released for cultivation.

➢ Water Resource: India has also taken steps to increase the optimal use of water resources, create infrastructure for irrigation, conserve soil fertility with balanced use of fertilizers, and provide connectivity from farms to markets.

➢ Indian Agri-Sector in Covid: Indian agriculture has achieved great success after the country's independence, and the sector also remained unaffected even during the Covid pandemic.

➢ India Resolved to: Continue working together to achieve the Poverty Reduction' and 'Zero Hunger Goal as part of the sustainable development goals. Cooperate in research and development as well as exchange of best practices to enhance productivity.

Related Indian Initiatives:

❖ The 'Per Drop-More Crop' scheme for irrigation and 'Paramparagat Krishi Vikas Yojana' for organic farming is being successfully implemented.

❖ Pradhan Mantri Fasal Bima Yojana is being implemented to provide insurance cover for the farmers.

❖ To address the malnutrition problem, India is running the world's largest food-based safety net programme, which includes the Public Distribution System and the Mid-Day Meal Scheme.

The government is also providing an annual income support of Rs 6,000 under the Pradhan Mantri Kisan Samman Nidhi (PM-KISAN).

1.7. CRITICALLY ANALYSE THE LACK OF ‘FRONT-OF-PACK LABELLING SYSTEM’ IN INDIA.

Context: The Food Safety and Standards Authority of India (FSSAI), in 2018, released the Draft Food Safety and Standards (Labelling and Display) Regulation. However, even after so many expert panel recommendations and regulations, India still does not have a clear labelling or Front-of-pack (FoP) Labelling System, which can warn consumers about harmful levels of fat, salt and sugar in processed foods.

Page 27: MONTHLY NEWS DIARY SEPTEMBER 2021

MONTHLY NEWS DIARY SEPTEMBER 2021

www.sosinclasses.com +91 90000 36699 [email protected]

Pag

e24

About Front-of-Pack (FoP) Labelling System:

• FoP labelling system has long been listed as one of the global best practices to nudge consumers into healthy food choices.

• It works just the way cigarette packets are labelled with images to discourage consumption.

• As India is experiencing a dietary shift, with people increasingly consuming more processed and ultra-processed foods and a burgeoning market, these factors prompt the need for FoP labelling for India.

• It will play a handy role in fighting increasing obesity and many non-communicable diseases.

• The World Health Organization (WHO) defines FoP labels as nutrition labelling systems that are presented on the front of food packages in the principal field of vision; and present simple, often graphic information on the nutrient content or nutritional quality of products.

• To complement the more detailed nutrient declarations provided on the back of food packages.

• The Codex Alimentarius Commission, an international food standards body established jointly by WHO and the Food and Agriculture organization (FAO), mentions that “FoP labelling is designed to assist in interpreting nutrient declarations”.

About the Draft Food Safety and Standards (Labelling And Display) Regulations: The rules mandates colour-coded labels on food items. The draft regulation has been brought to encourage consumers to make healthier food choices and inform them about what the product actually contains. The front of all packaged food items will have to display the total

Page 28: MONTHLY NEWS DIARY SEPTEMBER 2021

MONTHLY NEWS DIARY SEPTEMBER 2021

www.sosinclasses.com +91 90000 36699 [email protected]

Pag

e25

number of calories, saturated and trans fats, salt, and added sugar content as well as the proportion of the daily energy needs fulfilled by the food item. FSSAI has also changed the symbol of vegetarian food from a green circle to a green triangle to help color blind people distinguish it from the brown circle denoting non-vegetarian food. According to the proposed regulation, if the total amounts of calories, fats, trans-fats, sugar, and sodium per serving exceed the stipulated limits, it would be indicated in red colour.

Issues Pertaining to these Rules:

➢ Masking of Positive Nutrients: Most consumer organisations objected as ‘positive nutrients will mask the negative impact of high fat, salt and sugar in the food and the industry will use it to mislead the consumer.

➢ FSSAI proposed to also consider ‘positive nutrients’ in the FoP label. It was about giving scores to ‘positive nutrients’ such as proteins, nuts, fruits and vegetables in the name of promoting wholesome foods.

➢ Restricted Target Audience: The labelling format appears to be aimed only at individuals who are literate and nutritionally aware.

➢ Further, limited general and nutrition literacy mean understanding of the text-intensive nutrient information is difficult.

➢ Objections from Food Industry: Indian food industries have expressed many concerns over the proposed format, especially using the colour red as it indicates danger and could dissuade consumers from their products.

Way Forward

• More Focus on Pictorial Representation: Almost a quarter of the Indian population is illiterate, therefore pictorial representation would allow better engagement and understanding. It might be beneficial for front-of-pack labelling in India to be symbol based, with food images, logos, and health benefits. Mandatory front of pack labelling must be preceded by strong research and in a format that is understandable and acceptable to all.

1.8. WHAT ARE THE OBJECTIVES OF THE RAIL KAUSHAL VIKAS YOJANA UNDER PMKVY.

Background: Recently, the Ministry of Railways launched Rail Kaushal Vikas Yojana (RKVY)

under Pradhan Mantri Kaushal Vikas Yojana (PMKVY).

Rail Kaushal Vikas Yojana (RKVY)

➢ This is a skill development programme, where training will be provided to youth with a

special focus on jobs that are relevant to the Railways.

➢ The training will be provided in four trades viz. Electrician, Welder, Machinist and Fitter

and other trades will be added by zonal railways and Production units based on regional

demands and needs assessment.

➢ Training will be provided to apprentices under the Apprentice Act 1961.

Page 29: MONTHLY NEWS DIARY SEPTEMBER 2021

MONTHLY NEWS DIARY SEPTEMBER 2021

www.sosinclasses.com +91 90000 36699 [email protected]

Pag

e26

Objectives:

• To impart training skills to the youth in various trades to bring qualitative improvement.

• To train 50,000 candidates over the next three years.

▪ Eligibility: Candidates who are 10th passed and between 18-35 years shall be eligible to apply. Participants in the scheme shall however have no claim to seek employment in Railways on the basis of this training.

Significance: The scheme will not only improve the employability of the youth but also upgrade the skills of self-employed and those who are working with contractors through reskilling and up-skilling.

Pradhan Mantri Kaushal Vikas Yojana

➢ Launched in 2015, it is a flagship program of the Ministry of Skill Development and Entrepreneurship (MSDE).

➢ It aims to mobilize youth to take up skill training with the aim of increasing productivity and aligning the training and certification to the needs of the country.

➢ Owing to the success of PMKVY 1.0 wherein more than 19 lakh students were trained as against the target of 24 lakh, the scheme was relaunched as PMKVY 2.0 (2016-2020) with an allocated budget of Rs. 12000 Crores that aimed to train 10 million youth by the year 2020.

➢ And further PMKVY 3.0 was launched in 2021, in a bid to empower India’s youth with employable skills by making over 300 skill courses available to them. It envisages training of eight lakh candidates over a scheme period of 2020-2021 with an outlay of Rs. 948.90 crore.

Key Components:

✓ Short Term Training: Training as per National Skills Qualification Framework (NSQF) is provided to those who are either school/college dropouts or unemployed.

✓ Recognition of Prior Learning (RPL): An individual with a certain set of skills or with prior learning experience is assessed and certified under RPL with grade according to the NSQF.

✓ Special Projects: This component ensures training in special areas and premises of government bodies and corporate. It aims to encourage training in vulnerable and marginalized groups of society.

✓ Training Partners (TPs) are mandated to organize Kaushal and Rozgar Melas every six months, thus providing placement assistance to certified ones.

Page 30: MONTHLY NEWS DIARY SEPTEMBER 2021

MONTHLY NEWS DIARY SEPTEMBER 2021

www.sosinclasses.com +91 90000 36699 [email protected]

Pag

e27

1.9. EXAMINE THE FOOD SAFETY ECOSYSTEM IN INDIA UNDER THE PURVIEW OF FSSAI.

Background: Recently, the union minister for Health and Family Welfare has released the 3rd State Food Safety Index (SFSI). Also, 19 Mobile Food Testing Vans (Food Safety on Wheels) have been flagged off to supplement the food safety ecosystem across the country.

About the Index:

➢ The index is developed by FSSAI (Food Safety and Standards Authority of India) to measure the performance of states on five significant parameters of Food Safety.

➢ The parameters include Human Resources and Institutional Data, Compliance, Food Testing – Infrastructure and Surveillance, Training & Capacity Building and Consumer Empowerment.

➢ The Index is a dynamic quantitative and qualitative benchmarking model that provides an objective framework for evaluating food safety across all States/UTs.

➢ The first State Food Safety Index for the year 2018-19 was announced on the first-ever World Food Safety Day on 7th June 2019.

Importance of Food Safety:

• Access to sufficient amounts of safe food is key to sustaining life and promoting good health.

• Foodborne illnesses are usually infectious or toxic in nature and often invisible to the plain eye, caused by bacteria, viruses, parasites or chemical substances entering the body through contaminated food or water.

• An estimated 4,20,000 people around the world die every year after eating contaminated food and children under 5 years of age carry 40% of the foodborne disease burden, with 1,25, 000 deaths every year.

• Food safety has a critical role in assuring that food stays safe at every stage of the food chain - from production to harvest, processing, storage, distribution, all the way to preparation and consumption.

• Food production is responsible for up to 30% of global greenhouse-gas emissions contributing to global warming.

Related Initiatives:

Indian:

➢ Eat Right India Movement: ▪ It is an initiative of the Government of India and FSSAI to transform the country’s

food system in order to ensure safe, healthy and sustainable food for all Indians.

Ranking of States: Among the larger states, Gujarat was the top-ranking state, followed by Kerala and Tamil Nadu. Among the smaller states, Goa stood first followed by Meghalaya and Manipur. Among UTs, Jammu & Kashmir, Andaman & Nicobar Islands and New Delhi secured top ranks.

Page 31: MONTHLY NEWS DIARY SEPTEMBER 2021

MONTHLY NEWS DIARY SEPTEMBER 2021

www.sosinclasses.com +91 90000 36699 [email protected]

Pag

e28

▪ It is aligned to the National Health Policy 2017 with its focus on preventive and promotive healthcare and flagship programmes like Ayushman Bharat, POSHAN Abhiyaan, Anemia Mukt Bharat and Swachh Bharat Mission.

➢ Eat Right Station Certification: ▪ The certification is awarded by FSSAI to railway stations that set benchmarks (as

per the Food Safety and Standards Act, 2006) in providing safe and wholesome food to passengers.

▪ The Eat Right Research Awards and Grants to encourage and recognize high-quality research in the area of food safety and nutrition in India has also been launched.

▪ The results of PAN-India survey for identifying the presence of industrially produced trans fatty acid content in the selected foods has been released. Overall, only 84 samples, i.e. 1.34%, have more than 3% industrially produced trans fats from the total of 6245 samples.

▪ In an effort to engage industry on the issue of plastic in food packaging, 24 food businesses signed a pledge on becoming “Plastic Waste Neutral” by collecting, processing and recycling of 100% post-consumer plastic waste from across the sources.

Global:

➢ The Codex Alimentarius, or "Food Code" is a collection of standards, guidelines and codes of practice adopted by the Codex Alimentarius Commission.

➢ The Codex Alimentarius Commission is a joint intergovernmental body of the Food and Agriculture Organization (FAO) and World Health Organization (WHO).

➢ Currently, it has 189 members and India is a member.

1.10. ASSESS THE IMPLEMENTATION OF THE FOREST RIGHTS ACT (FRA) IN INDIA.

Background: Recently, the Jammu and Kashmir government has decided to implement the Forest Rights Act (FRA), 2006, which will elevate the socio-economic status of a sizable section of the 14-lakh-strong population of tribals and nomadic communities.

About FRA FRA enacted in 2006 recognises the rights of forest-dwelling tribal communities

and other traditional forest dwellers to forest resources on which these communities were dependent for a variety of needs, including livelihood, habitation, and other sociocultural needs.

It recognizes and vest the forest rights and occupation in Forest land in Forest Dwelling Scheduled Tribes (FDST) and Other Traditional Forest Dwellers (OTFD) who have been residing in such forests for generations.

It strengthens the conservation regime of the forests while ensuring livelihood and food security of the FDST and OTFD.

The Gram Sabha is the authority to initiate the process for determining the nature and extent of Individual Forest Rights (IFR) or Community Forest Rights (CFR) or both that may be given to FDST and OTFD.

Page 32: MONTHLY NEWS DIARY SEPTEMBER 2021

MONTHLY NEWS DIARY SEPTEMBER 2021

www.sosinclasses.com +91 90000 36699 [email protected]

Pag

e29

Rights Under the Forest Rights Act:

➢ Title rights: It gives FDST and OTFD the right to ownership to land farmed by tribals or forest dwellers subject to a maximum of 4 hectares. Ownership is only for land that is actually being cultivated by the concerned family and no new lands will be granted.

➢ Use rights: The rights of the dwellers extend to extracting Minor Forest Produce, grazing areas etc.

➢ Relief and development rights: To rehabilitate in case of illegal eviction or forced displacement and to basic amenities, subject to restrictions for forest protection.

➢ Forest management rights: It includes the right to protect, regenerate or conserve or manage any community forest resource which they have been traditionally protecting and conserving for sustainable use.

Significance:

➢ Constitutional Provision Expansion: It expands the mandate of the Fifth and the Sixth Schedules of the Constitution that protect the claims of indigenous communities over tracts of land or forests they inhabit.

➢ Security Concerns: The alienation of tribes was one of the factors behind the Naxal Movement, which affected states like Chhattisgarh, Odisha and Jharkhand.

➢ Forest Governance: It has the potential to democratise forest governance by recognising community forest resource rights. It will ensure that people get to manage their forest on their own, which will regulate exploitation of forest resources by officials, improve forest governance and better management of tribal rights.

Challenges:

➢ Administrative Apathy: As tribals are not a big vote bank in most states, governments find it convenient to subvert FRA or not bother about it at all in favour of monetary gains. The forest bureaucracy has misinterpreted the FRA as an instrument to regularise encroachment instead of a welfare measure for tribals. Corporates fear they may lose the cheap access to valuable natural resources.

➢ Dilution of Act: Certain sections of environmentalists raise the concern that FRA bends more in the favour of individual rights, giving lesser scope for community rights.

➢ Institutional Roadblock: Rough maps of community and individual claims are prepared by Gram Sabha which at times often lack technical knowhow and suffers from educational incapacity.

Misuse of FRA: The FRA has been misused and communities have rushed to file claims. Politicians across party lines have interpreted FRA as a land distribution exercise and have fixed targets for districts.

Way Forward:

It is important that the governments at Central and State levels are strengthened with human and financial resources to help implement FRA on a mission mode. Besides leveraging modern technology to map and monitor the implementation of FRA, the forest bureaucracy must also be reformed to serve as service providers to gram sabhas.

Page 33: MONTHLY NEWS DIARY SEPTEMBER 2021

MONTHLY NEWS DIARY SEPTEMBER 2021

www.sosinclasses.com +91 90000 36699 [email protected]

Pag

e30

2. INTERNATIONAL RELATIONS

2.1. ASSESS THE SIGNIFICANCE OF THE RESOLUTION 2593 ON TALIBAN.

Context: Recently, the India-led United Nations Security Council (UNSC) adopted a Resolution 2593 on Taliban. The resolution, sponsored by France, UK and the US, was adopted with 13 members, including India, voting in favour, none against it. Two permanent and veto-wielding members Russia and China abstained. The adoption of the resolution is a strong signal from the Security Council and the international community on its expectations in respect of Afghanistan.

Russia and China’s Abstention:

Russia abstained as the resolution wasn’t specific enough about terror threats, did not speak of the “brain drain” effect of evacuating Afghans and did not address the economic and humanitarian consequences of US freezing the Afghan government’s US accounts following the Taliban takeover.

China shared some of Russia’s concerns. It believes that the current chaos was a direct consequence of Western countries’ “disorderly withdrawal”.

China is of the view that it is necessary for the international community to engage with the Taliban, and actively provide them with guidance.

Russia and China wanted all the terrorist groups, especially Islamic State (ISIS) and the Uighur East Turkestan Islamic Movement (ETIM) to be named specifically in the document.

India’s recent steps:

➢ India has directed a high-level group composed of EAM (External Affairs Minister), NSA (National Security Adviser) and senior officials to focus on the immediate priorities of India.

➢ The group is seized of issues pertaining to the safe return of stranded Indians to India and is assuring that the territory of Afghanistan is not used in any manner for terrorism directed against India.

➢ Recently, India’s ambassador to Qatar met with the head of the Taliban’s political office. It is the first time the government has publicly acknowledged such a meeting which came at the request of the Taliban. The Taliban leader assured that all the issues would be positively addressed.

What is Resolution 2593? It reiterates the importance of combating terrorism in Afghanistan, including those individuals and entities designated pursuant to resolution 1267 (1999). The resolution called for the Taliban to facilitate safe passage for people wanting to leave Afghanistan, allow humanitarians to access the country, uphold human rights, including for women and children and inclusive and negotiated political settlement.

Page 34: MONTHLY NEWS DIARY SEPTEMBER 2021

MONTHLY NEWS DIARY SEPTEMBER 2021

www.sosinclasses.com +91 90000 36699 [email protected]

Pag

e31

Afghanistan’s Representation in Multilateral Organizations:

With uncertainty hanging over the international representation of Afghanistan under the Taliban, a question has risen over the membership of the country in the South Asian Association for Regional Cooperation (SAARC).

The question on representation of Afghanistan at the SAARC has come up especially since a similar issue is yet to be addressed by the United Nations.

SAARC is already facing many issues and the current situation of Afghanistan has further increased the problems for it.

Afghanistan was admitted into the SAARC as the eighth member in 2007.

Conventionally, countries do not lose membership of regional or global platforms because of a domestic political change.

However, a similar question is also likely to come up in the Kathmandu-based intergovernmental organisation the International Centre for Integrated Mountain Development (ICIMOD).

Way Forward:

• India is expected to chair the 1988 Sanctions committee that looks at Taliban sanctions next and participate in the decision to extend the mandate of the UN Assistance Mission in Afghanistan (UNAMA), where it will also have to balance competing demands from the US, the UK and France bloc against Russia and China.

• India’s Afghan policy is at a major crossroads; to safeguard its assets there as well as to stay relevant in the unfolding ‘great game’ in and around Afghanistan, India must reset its Afghanistan policy accordingly.

2.2. WHAT IS THE SIGNIFICANCE OF THE CHINA-MYANMAR NEW PASSAGE? AND ASSESS ITS IMPACT ON INDIA.

The shipments on a newly launched railway line under the China-Myanmar New Passage from the Myanmar border to the key commercial hub of Chengdu in western China have started.

China-Myanmar New Passage

➢ The passage provides China a new road-rail transportation channel to the Indian Ocean, were delivered last week, state media reported on Tuesday.

➢ The transport corridor involves a sea-road-rail link.

➢ It connects the logistics lines of Singapore, Myanmar, and China, and is currently the most convenient land and sea channel linking the Indian Ocean with southwest China.

Page 35: MONTHLY NEWS DIARY SEPTEMBER 2021

MONTHLY NEWS DIARY SEPTEMBER 2021

www.sosinclasses.com +91 90000 36699 [email protected]

Pag

e32

➢ Goods from Singapore reached Yangon Port, arriving by ship through the Andaman Sea of the northeastern Indian Ocean, and were then transported by road to Lincang on the Chinese side of the Myanmar-China border in Yunnan province.

➢ The new railway line that runs from the border town of Lincang to Chengdu, a key trade hub in western China, completes the corridor.

Impact on India:

• From the perspective of security, India’s border with Myanmar has historically presented serious security challenges.

• Chinese troops had used the Myanmar route to threaten India’s North-eastern States prior to the 1962 war.

• In the run-up to the India-China war of 1962, Chinese troops had commissioned local muleteers in Northern Myanmar to facilitate the movement of troops and war logistics to challenge India’s Northeast.

Way forward:

The work on infrastructure projects in India’s North-eastern States needs to be expedited to ensure speedy mobilization of India’s own troops to face different contingencies. Monitoring of developments including deployment of space assets to ensure that India is not caught unaware would be desirable. Most importantly, India on its part needs to substantially step up its own game in Myanmar and proactively engage Myanmar in the realm of the infrastructure upgrade.

Page 36: MONTHLY NEWS DIARY SEPTEMBER 2021

MONTHLY NEWS DIARY SEPTEMBER 2021

www.sosinclasses.com +91 90000 36699 [email protected]

Pag

e33

2.3. AS INDIA-AUSTRALIA FIRST 2+2 DIALOGUE WAS HELD, EXAMINE ITS IMPACT ON INDIA.

Background:

Recently, the foreign and defence ministers of India and Australia held the first India-Australia 2+2 Ministerial Dialogue in New Delhi. The inaugural dialogue is held in pursuant to the elevation of India-Australia bilateral relations to comprehensive strategic partnership during the India-Australia Leaders’ virtual summit in 2021.

Focus on Indo-Pacific: To maintain an open, free, prosperous, and rules-based Indo-Pacific region (in line with the United Nations Convention on the Law of the Sea (UNCLOS)). To support India’s Indo-Pacific Oceans’ Initiative. Need for renewed efforts by the Quad member countries to expand cooperation in the region.

Focus on Supply Chain Resilience Initiative: To work together through multilateral, regional and plurilateral mechanisms to diversify supply chains between trusted and reliable trading partners for critical health, technology and other goods and services. In this context, they welcomed the launch of the Supply Chain Resilience Initiative by the Trade Ministers of India, Australia and Japan.

Keeping-up with the Momentum: Decided to meet at least once every two years in this format to keep up the momentum.

Shared Vision on Afghanistan: Displayed a common approach to the Afghan crisis, after the recent Taliban takeover of Afghanistan. India held that the policy is summed up by the United Nations Security Council Resolution 2593. UNSC Resolution 2593, emphasises most of all, that Afghanistan must not allow its soil to be used in any manner by any body for terrorism.

Countering Terrorism: To continue to work together on counter terrorism; countering of radicalization and on the proposed UN Comprehensive Convention on International Terrorism.

Page 37: MONTHLY NEWS DIARY SEPTEMBER 2021

MONTHLY NEWS DIARY SEPTEMBER 2021

www.sosinclasses.com +91 90000 36699 [email protected]

Pag

e34

Strengthening Bilateral Relationship: Discussed strengthening their relationship in bilateral trade, vaccines, defence production, community links, maritime security, cyber and climate cooperation.

Cooperation on Covid-19: Agreement was reached to deepen cooperation in vaccine manufacturing, including under the Quad framework, and to deliver high quality vaccines to their Indo-Pacific partners. Researchers of both countries are working together to advance Covid-19 screening and study the future health effects of the virus through projects funded by the Australia-India Strategic Research Fund.

Defence Relationship: Australia has invited India for participation in future Talisman Sabre exercises which will raise interoperability while both sides explore longer term reciprocal arrangements in logistics support.

Economic Agreements: There was renewed support expressed for finalization of bilateral Comprehensive Economic Cooperation Agreement. Apart from this, both countries also pitched for early resolution of the issue of taxation of offshore income of Indian firms under the India Australia Double Taxation Avoidance Agreement.

Others: While reaffirming support for the Indian candidacy for permanent membership of the UNSC, grants of Australian $ 1 million to the International Solar Alliance, and Australian $10 million to the Coalition for Disaster Resilient infrastructure (both India-led initiatives).

India-Australia Relations:

➢ Geo-Political Relations: Over the past few years, China’s actions– including extensive island building in the South China Sea has heightened concerns in a number of countries around the world. This has led to formation of Quad (India, Australia, Japan and US) grouping.

➢ Defence Relations: Bilateral naval exercise (AUSINDEX), Ex AUSTRA HIND (Bilateral Exercise with Army), EX PITCH BLACK (Australia’s multilateral air combat training exercise) and multilateral Malabar Exercise (comprising Quad countries). The countries have signed the Mutual Logistic Support Arrangement (MLSA).

➢ Multilateral Cooperation: Both are members of the Quad, Commonwealth, Indian Ocean Rim Association (IORA), ASEAN Regional Forum, Asia Pacific Partnership on Climate and Clean Development, and have participated in the East Asia Summits. Both countries have also been cooperating as members of the Five Interested Parties (FIP) in the World Trade Organization context. Australia is an important player in Asia Pacific Economic Cooperation (APEC) and supports India's membership of the organisation.

2+2 Dialogue 2+2 Ministerial is the highest-level

institutional mechanism between the two countries.

It is a format of dialogue where the defence/foreign ministers or secretaries meet with their counterparts from another country.

India holds such talks with Australia, Japan and the USA.

Page 38: MONTHLY NEWS DIARY SEPTEMBER 2021

MONTHLY NEWS DIARY SEPTEMBER 2021

www.sosinclasses.com +91 90000 36699 [email protected]

Pag

e35

• Other Diplomatic Engagements: A Civil Nuclear Cooperation Agreement was signed in September 2014. The Mutual Legal Assistance Treaty (MLAT) and the Extradition Treaty, which were signed in June 2008. Further, recently India-Australia Circular Economy Hackathon (I-ACE) was also organized.

2.4. ELABORATE THE OBJECTIVES OF US-INDIA STRATEGIC CLEAN ENERGY PARTNERSHIP (SCEP).

Background: Recently, the revamped US-India Strategic Clean Energy Partnership SCEP was

launched during the ministerial meeting of the Petroleum and Natural Gas Ministry with the US

Ministry of Energy. The SCEP was launched in accordance with the US - India Climate and Clean

Energy Agenda 2030 Partnership announced by both countries at the Leaders’ Summit on

Climate held earlier this year (2021).

Revamped Strategic Clean Energy Partnership (SCEP):

✓ Addition of a fifth Pillar on

Emerging Fuels (cleaner

energy fuels).

✓ With this, the SCEP inter-

governmental engagement

now spreads across five

pillars of cooperation - Power

and Energy Efficiency,

Responsible Oil and Gas,

Renewable Energy,

Sustainable Growth and

Emerging Fuels.

✓ Support India in achieving its goal of installing 450GW of renewable energy capacity by

2030.

✓ A new India-U.S. Task Force on Biofuels was also announced.

Reviewed India-US Civil Nuclear Energy Cooperation:

▪ The Indo-US nuclear cooperation agreement was signed in 2008.

▪ A major aspect of the Deal was that the Nuclear Suppliers Group (NSG) gave a special

waiver to India that enabled it to sign cooperation agreements with a dozen countries.

Transformation of the Gas Task Force: It will be the India-US Low Emissions Gas Task Force. It

will focus on addressing India’s natural gas policy, technology, and regulatory barriers by

promoting efficient and market-driven solutions aimed at meeting India’s growing energy

demand, and greenhouse gas emission reduction targets.

US-India Agenda 2030 Partnership: ▪ The aim is to create stronger bilateral

cooperation on actions in the current decade to meet the goals of the Paris Agreement.

▪ The Partnership will proceed along two main tracks: the Strategic Clean Energy Partnership and the Climate Action and Finance Mobilization Dialogue.

▪ India elevated the India-US energy dialogue to a strategic energy partnership in 2018.

Page 39: MONTHLY NEWS DIARY SEPTEMBER 2021

MONTHLY NEWS DIARY SEPTEMBER 2021

www.sosinclasses.com +91 90000 36699 [email protected]

Pag

e36

Institutionalism of India Energy Modeling Forum: Six Task Forces were constituted for carrying

out research and modelling in different areas. There will be deliberations on Energy Data

Management, Low Carbon Technologies and Just Transition in the Coal Sector.

Expanded the Scope of (PACE)-R Initiative: Agreed to include smart grid and grid storage as

part of the second phase of the Partnership to Advance Clean Energy (PACE)-R initiative

anchored on the Indian side by the Department of Science & Technology.

Recent Updates on US-India Relations:

➢ Malabar Exercise: The Navies of the Quad (Quadrilateral Framework) Nations (India, the

United States, Japan, and Australia) participated in the 25th edition of the exercise.

➢ India-US Agreement on ALUAV: India and the US have signed a Project Agreement (PA)

to jointly develop an Air-launched Unmanned Aerial Vehicle (ALUAV) or drones that can

be launched from an aircraft.

➢ Issues in Free Trade Agreement: The US administration has indicated that it is no longer

interested in securing a bilateral Free Trade Agreement (FTA) with India.

➢ NISAR: NASA and ISRO are collaborating on developing an SUV-sized satellite called

NISAR, which will detect movements of the planet’s surface as small as 0.4 inches over

areas about half the size of a tennis court.

2.5. EXPLAIN THE PILLARS OF CLIMATE ACTION AND FINANCE MOBILISATION DIALOGUE AND ITS SIGNIFICANCE TO INDIA.

Context: Recently, US Special Presidential Envoy for Climate, along with India’s Union Minister for Environment, Forests and Climate Change launched the Climate Action and Finance Mobilisation Dialogue (CAFMD) between the two countries. India has so far abstained from committing to a net zero goal but is on a climate pathway that is compatible with keeping global temperatures to below 2 degrees Celsius by the end of the century. Pillars to the CAFM:

➢ Climate Action Pillar: It would have joint proposals looking at ways in which emissions could be reduced in the next decade.

➢ Finance Pillar: Through this the US will

CAFMD It is one of the two tracks of the India-US Climate and Clean

Energy Agenda 2030 partnership launched at the Leaders' Summit on Climate in April 2021.

Earlier, the revamped US-India Strategic Clean Energy Partnership SCEP (first track) was launched.

It will provide both countries an opportunity to renew collaborations on climate change while addressing financing aspects and deliver climate finances primarily as grants and concessional finance as envisaged under the Paris Agreement.

It will also help to demonstrate how the world can align swift climate action with inclusive and resilient economic development, taking into account national circumstances and sustainable development priorities.

Page 40: MONTHLY NEWS DIARY SEPTEMBER 2021

MONTHLY NEWS DIARY SEPTEMBER 2021

www.sosinclasses.com +91 90000 36699 [email protected]

Pag

e37

collaborate in attracting capital and enhancing the enabling environment to deploy 450 GW of renewable energy capacity in India and demonstrate and scale innovative clean energy technologies and promote bilateral clean energy investment and trade.

➢ Adaptation and Resilience: The two countries will collaborate in building capacities to “measure and manage climate risks”.

Opportunities for India:

There’s never been a better time to invest in the energy transition. Renewable energy is cheaper than ever.

In fact, it is cheaper to build a solar farm in India than anywhere else on the planet. Investors are now moving to clean energy all around the world and the energy transition

is already rebounding after the worst of the pandemic and is now on track to smash the pre-pandemic record of 8.4 billion USD invested in one year.

The International Energy Agency forecasts that if India seizes the clean energy opportunity, it could become the world’s largest market for batteries and solar panels.

Currently India's installed power capacity is projected to be 476 GW by 2021-22 and is expected to rise to at least 817 GW by 2030.

2.6. EXAMINE THE CONNECTIVITY PROJECTS OF INDIA AND SOUTHEAST ASIA REGION.

Background: Recently, at the virtual ASEAN (Association of Southeast Asian Nations) summit on Future of India-ASEAN Connectivity Partnerships, the Indian government underlined the importance of cross-border connectivity between India and Southeast Asian nations. ASEAN brings together ten Southeast Asian states – Brunei, Cambodia, Indonesia, Laos, Malaysia, Myanmar, the Philippines, Singapore, Thailand and Vietnam – into one organisation.

Connectivity between India and Southeast Asia:

➢ India currently is working with ASEAN on multiple connectivity projects, through land, water, and air.

➢ Giving substance to ASEAN-India relations through connectivity will gradually change the geopolitical landscape of this region.

➢ In this context, India has now been actively developing infrastructure in Northeastern India. Bangladesh-China-India-Myanmar (BCIM) Corridor is part of this.

➢ These connectivity projects will not only curb existing insurgencies, but also help India’s northeastern states to develop their economic potential and integrate with mainland India.

➢ Further, India-ASEAN Free Trade Agreement (FTA) is central to India’s growing engagement with her eastern neighbours.

➢ It will enable small and medium-sized enterprises in the border areas to explore new business opportunities.

Page 41: MONTHLY NEWS DIARY SEPTEMBER 2021

MONTHLY NEWS DIARY SEPTEMBER 2021

www.sosinclasses.com +91 90000 36699 [email protected]

Pag

e38

Examples of Cross Connectivity Projects:

❖ India-Myanmar-Thailand Trilateral Highway: One of the flagship projects for land connectivity between ASEAN and India. Connecting the three countries from Moreh in India to Mae Sot in Thailand to Bagan in Myanmar, was first proposed in 2002. Besides this, the India, Myanmar, and Thailand Motor Vehicles Agreement (IMT MVA) is in the final stage. On realization, this will become the first-ever cross border facilitation agreement between South and Southeast Asia.

❖ Kaladan Multi-Modal Transit Transport Project (KMMTTP): To develop connectivity through water, ASEAN and India are working on KMMTTP. Initiated by the Indian government in 2008 and is entirely funded by India. To seek an alternative route through Myanmar for the transportation of goods to the northeast region of India. Connects Kolkata in India to Sittwe and Paletwa in Myanmar by sea and river respectively. Alongside this initiative, India has contributed to build a sea link via Sittwe port in Myanmar in order to enable an alternative transit route through Bangladesh.

❖ Mekong-India Economic Corridor (MIEC): Involves the integration of four Mekong countries – Vietnam, Myanmar, Thailand, and Cambodia - along with India, connecting Ho Chi Minh City, Dawei, Bangkok, and Phnom Penh with Chennai. The corridor will provide opportunities for the participant countries to develop infrastructure, enhance their economic base with the region, and especially reduce the transit distance between India and ASEAN countries.

Page 42: MONTHLY NEWS DIARY SEPTEMBER 2021

MONTHLY NEWS DIARY SEPTEMBER 2021

www.sosinclasses.com +91 90000 36699 [email protected]

Pag

e39

Way Forward

• Extension of Trilateral Highway: The Trilateral Highway can be extended to Cambodia, Laos and Vietnam. This will enable greater connectivity and economic integration of India's northeast with its eastern neighbours.

• Digital Highway: As such beyond movement of goods and physical connectivity, it is also important for two regions to explore ways to enhance digital connectivity. This is in concurrence with the Government of India efforts to turn India into a Global Data Hub.

• Improving Maritime Connectivity: With the introduction of the “Sagarmala” project, India plans to invest in port infrastructure for improved integration and connectivity through the sea. This is an encouraging step towards enhancing India-ASEAN Connectivity projects.

2.7. WHAT IS THE SIGNIFICANCE OF AUKUS GROUPING?

Background: Recently, the US has announced a new trilateral security partnership for the Indo-Pacific, between Australia, the UK and the US (AUKUS). The major highlight of this arrangement is the sharing of US nuclear submarine technology to Australia.

Focusing on Indo-Pacific: The security grouping AUKUS will focus on advancing strategic interests in the Indo-Pacific region. Though the US has explicitly denied that the grouping is targeted against China, its Indo-pacific orientation makes it an alliance against China’s assertive actions in the South China Sea. The three countries alongside Canada and New Zealand already share extensive intelligence through the Five Eyes alliance.

Transferring Nuclear Submarines to Australia:

▪ As part of this initiative, Australia will acquire nuclear-powered submarines with help from the UK and the US.

▪ This step is significant because the US has only shared nuclear submarine technology once before, and it started in 1958 with Great Britain.

▪ Nuclear submarines that are quieter than their conventional counterparts but also more capable of being deployed for longer periods and needing to surface less frequently.

Page 43: MONTHLY NEWS DIARY SEPTEMBER 2021

MONTHLY NEWS DIARY SEPTEMBER 2021

www.sosinclasses.com +91 90000 36699 [email protected]

Pag

e40

▪ Alongside India’s stated intent to acquire more nuclear-powered submarines, it will amount to a step-change increase in the Quad’s undersea and anti-submarine warfare capabilities. Quad is a grouping of India, USA, Australia and Japan.

▪ Australia is now set to join an elite group of only six countries – India, the US, the UK, France, Russia and China – that operate nuclear-powered submarines. It will also be the only country to have such submarines without having a civilian nuclear power industry.

Multi-Sectoral Cooperation: AUKUS will also involve a new architecture of meetings and engagements between the three countries, as well as cooperation across emerging technologies (applied AI, quantum technologies and undersea capabilities).

Five Eyes Alliance

The Five Eyes alliance is an intelligence-sharing arrangement between five English-speaking democracies: the US, UK, Canada, Australia and New Zealand.

The alliance was created during the Cold War (1946-1991) that was fought between the United States and the Soviet Union, as well as their respective allies.

The alliance was needed to share sensitive information regarding their adversaries on all possible fronts available. It is often described as the world's most successful intelligence alliance.

2.8. Analyse the objectives of the 15TH East Asia Summit Energy Ministers Meeting.

Background: Recently, the Union Minister of State for Power attended the 15th East Asia Summit Energy Ministers Meeting. The theme of the meeting was “We Care, We Prepare, We Prosper”.

About the Meeting:

➢ The meeting was aimed at coordinating efforts of Association of Southeast Asian Nations (ASEAN) countries in pursuing the goal of energy security and energy transition that will benefit the people of the region.

➢ India confirmed that ASEAN is a region of great importance and engagement with ASEAN has been, and will remain, a critical element of India’s ‘Act East’ policy and strategy.

➢ Acting East is now a central element in India's Indo-Pacific vision. ➢ A brief status of the Indian energy transition plans, policies, challenges and efforts

towards decarbonisation was also provided. ➢ Some of India’s initiatives include National Solar Mission (NSM), Pradhan Mantri Ujjwala

Yojana (PMUY), Unnat Jyoti by Affordable LEDs for All (UJALA), Smart City Mission (SCM), etc.

Page 44: MONTHLY NEWS DIARY SEPTEMBER 2021

MONTHLY NEWS DIARY SEPTEMBER 2021

www.sosinclasses.com +91 90000 36699 [email protected]

Pag

e41

East Asia Summit:

About:

Established in 2005, it is a forum of 18 regional leaders for strategic dialogue and cooperation on the key political, security, and economic challenges facing the Indo-Pacific region.

The concept of an East Asia Grouping was first promoted in 1991 by the then Malaysian Prime Minister, Mahathir bin Mohamad.

There are six priority areas of regional cooperation within the framework of the EAS. These are – Environment and Energy, Education, Finance, Global Health Issues and Pandemic Diseases, Natural Disaster Management, and ASEAN Connectivity.

Membership:

It comprises the ten member states of the ASEAN which are Brunei, Cambodia, Indonesia, Laos, Malaysia, Myanmar, the Philippines, Singapore, Thailand, and Vietnam, along with 8 other countries namely Australia, China, Japan, India, New Zealand, the Republic of Korea, Russia and the USA. It is an ASEAN-centred forum so it can only be chaired by an ASEAN member. Brunei Darussalam is the chair for 2021.

EAS Meetings and Processes:

The EAS calendar culminates in the annual Leaders' Summit, which is usually held alongside ASEAN Leaders' meetings in the fourth quarter of every year. Meetings of EAS Foreign Ministers and Economic Ministers are also held annually.

India and EAS:

India is one of the founding members of the East Asia Summit. At the East Asia Summit in Bangkok in November 2019, India had unveiled India’s Indo-Pacific Oceans Initiative (IPOI), which is aimed at forging partnerships to create a secure and stable maritime domain.

Page 45: MONTHLY NEWS DIARY SEPTEMBER 2021

MONTHLY NEWS DIARY SEPTEMBER 2021

www.sosinclasses.com +91 90000 36699 [email protected]

Pag

e42

Other Related Groupings:

➢ ASEAN Defence Ministers Meeting (ADMM) Plus: It is an annual meeting of Defence Ministers of 10 ASEAN countries and eight dialogue partner countries. The ADMM-Plus countries include ten ASEAN Member States and eight Plus countries, namely Australia, China, India, Japan, New Zealand, Republic of Korea, Russian Federation, and the United States (same as in the EAS).

ASEAN Regional Forum: Established in 1994, the ASEAN Regional Forum (ARF) is an important platform for security dialogue in the Indo-Pacific. It comprises 27 members: the 10 ASEAN member states, the 10 ASEAN dialogue partners (Australia, Canada, China, the European Union, India, Japan, New Zealand, the Republic of Korea (ROK), Russia and the United States); Bangladesh, the Democratic People's Republic of Korea, Mongolia, Pakistan, Sri Lanka, and Timor-Leste; and one ASEAN observer (Papua New Guinea).

Page 46: MONTHLY NEWS DIARY SEPTEMBER 2021

MONTHLY NEWS DIARY SEPTEMBER 2021

www.sosinclasses.com +91 90000 36699 [email protected]

Pag

e43

4. PRELIMS WORKBOOK 1. With reference to Bye-elections, which of the following statements is/are correct?

1) It should be held within a period of six months from the date of the occurrence of the vacancy

2) Elections need not be conducted if the remainder of the term of a member in relation to a vacancy is less than one year Options:

a) 1 only b) 2 only c) Both d) None 2. Consider the following pairs:

Operation Objective

1) Samudra Setu – National effort to repatriate Indian citizens from

overseas during the COVID-19 pandemic.

2) Pawan – Military operation undertaken by the IPKF to take

control of the Jaffna Peninsula.

3) Maitri – Rescue and relief mission in quake-hit Nepal. Which of the above pairs is/are correctly matched?

a) 1 and 2 only b) 2 only c) 2 and 3 only d) 1, 2 and 3 3. Consider the following statements:

1) It is a landlocked country.

2) It is bordered by Russia to the east and northeast, and by Ukraine to the south.

3) It is not a member of the European Union.

The above statements describe:

a) Poland b) Slovenia c) Belarus d) Lithuania 4. Which of the following statements are incorrect?

1) The United Nations High Commissioner for Refugees (UNHCR) is a UN agency mandated to aid and protect refugees, forcibly displaced communities, and stateless people.

2) It was created in 1950 to address the refugee crisis that resulted from World War II. 3) The 1951 Refugee Convention established the scope and legal framework of the agency’s

work, which initially focused on Europeans uprooted by the war. 4) India is not a party to the 1951 Refugee Convention.

Options: a) 2 only b) 2 and 3 only c) 4 only d) None of the above 5. Which constitutional provision provides for reservation of seats in Panchayats?

a) Article 15 b) Article 239A c) Article 243D d) Article 280 6. What does the ‘Vidyanjali 2.0 portal’ provide for?

a) Online learning support and study resources to adults and illiterates. b) Scholarships for Indian students studying in elite universities abroad. c) Facilitates private donors, CSR contributions and volunteering activities for school

development. d) Creates a platform for teachers to form a community and share best practices.

Page 47: MONTHLY NEWS DIARY SEPTEMBER 2021

MONTHLY NEWS DIARY SEPTEMBER 2021

www.sosinclasses.com +91 90000 36699 [email protected]

Pag

e44

7. Consider the following statements with respect to PMGDISHA Scheme: 1) It is applicable only to rural areas of the country. 2) The implementation of the Scheme would be carried out under the overall supervision of

the Ministry of Rural Development. Which of the above statements is/are correct?

a) 1 only b) 2 only c) Both d) None 8. With reference to National Commission for Minorities (NCM), which of the following

statements is/are correct? 1) The Constitution of India defines the word ‘Minority’ on the basis of religion. 2) Five religious communities are notified as minority communities. 3) Each Member of the commission holds office for a period of three years from the date of

assumption of office. Options:

a) 1 only b) 1 and 2 only c) 3 only d) None 9. Consider the following statements with respect to ATL Space Challenge 2021: 1) It is an initiative of Atal Innovation Mission (AIM), ISRO, NASA and ESA. 2) The challenge will allow private players to focus on building new technologies for the space

sector Which of the above statements is/are correct?

a) 1 only b) 2 only c) Both d) None 10. With reference to National Pharmaceutical Pricing Authority (NPPA), which of the

following statements is/are correct? 1) NPPA is an independent body of experts under the Ministry of Health and Family Welfare. 2) NPPA is required to monitor the prices of drugs and take corrective measures where

warranted and has the power to fix and regulate such prices. Options:

a) 1 only b) 2 only c) Both d) None 11. Recently the term ‘Prompt Corrective Action (PCA)’ was seen in the news. What does it

refer to? a) PCA is a framework under which banks with weak financial metrics are put under watch by

the RBI.

b) A weather monitoring system deployed by India Meteorological Department. c) A new strategy against Maoists. d) An initiative in the National Education Policy to revamp the elementary education. 12. Consider the following statements with respect to Right to Information: 1) Only an individual who is above 18 years of age is eligible to seek information. 2) The right to information has been recognized as a fundamental right.

Which of the above statements is/are INCORRECT? a) 1 only b) 2 only c) Both d) None 13. Under the Constitution, a person shall be disqualified as being a member of the State

legislative assembly in which of the following cases? 1) If he is found guilty of certain election offences or corrupt practices in the elections.

Page 48: MONTHLY NEWS DIARY SEPTEMBER 2021

MONTHLY NEWS DIARY SEPTEMBER 2021

www.sosinclasses.com +91 90000 36699 [email protected]

Pag

e45

2) If he is of unsound mind and stands so declared by a court. 3) If punished for preaching and practising social crimes such as untouchability, dowry and

sati. 4) If he holds any office of profit under the Union or state government (except that of a

minister or any other office exempted by state legislature). Options:

a) 1 and 3 only b) 1, 2 and 3 only c) 2 and 4 only d) 1, 2, 3 and 4 14. With respect to anti-defection Law, which of the following statements is/are correct? 1) Under the Anti-defection Law, the power to decide the disqualification of an MP or MLA

rests with the presiding officer of the legislature in consultation with the Election Commission of India.

2) Supreme Court has observed that anti-defection cases should be decided by Speakers in six months’ time in all circumstances. Options:

a) 1 only b) 2 only c) Both d) None 15. Which of the following statements is/are correct? 1) The Cauvery River dispute is between Kerala, Karnataka, Tamil Nadu and Puducherry. 2) The sharing of the waters should be as per the Cauvery Water Disputes Tribunal’s decision

of 2007, which was modified by the Supreme Court in 2018. 3) The Central Government has notified the Cauvery Water Management Scheme which

constitutes the ‘Cauvery Water Management Authority’ and the ‘Cauvery Water Regulation Committee’ to give effect to the court’s decision. Options:

a) 1 only b) 1 and 2 only c) 2 and 3 only d) 1, 2 and 3 16. The Taiwan Strait connects: a) Strait of Malacca with Gulf of Thailand b) South China Sea with East China Sea c) Andaman Sea with Gulf of Thailand d) Indian Ocean with Pacific Ocean 17. Which of the following correctly describes the mandate of the Justice A.K. Rajan

committee that was recently constituted? a) To recommend a policy to the MOEFCC for clearing illegal encroachments in wetlands b) To study the impact of NEET on medical admissions in Tamil Nadu c) To suggest changes to the farm laws d) To investigate financing routes of Naxals in Dantewada and Bastar. 18. Consider the following statements with regard to PM Digital Health Mission: 1) It was launched in 2016 to augment India’s digital health infrastructure. 2) It includes a unique health ID for every citizen that will also work as their health account, to

which personal health records can be linked. 3) It will enable access and exchange of longitudinal health records of citizens with their

consent. Which of the above statements is/are incorrect?

a) 1 only b) 1 and 2 only c) 1 and 3 only d) None of the above

Page 49: MONTHLY NEWS DIARY SEPTEMBER 2021

MONTHLY NEWS DIARY SEPTEMBER 2021

www.sosinclasses.com +91 90000 36699 [email protected]

Pag

e46

19. Which country recently became the first European country to elect a women-majority Parliament?

a) France b) Norway c) Sweden d) Iceland 20. Hamboting La, recently seen in News is located in: a) Sikkim b) Arunachal Pradesh c) Ladakh d) Himachal Pradesh

Page 50: MONTHLY NEWS DIARY SEPTEMBER 2021

MONTHLY NEWS DIARY SEPTEMBER 2021

www.sosinclasses.com +91 90000 36699 [email protected]

Pag

e47

Page 51: MONTHLY NEWS DIARY SEPTEMBER 2021

MONTHLY NEWS DIARY SEPTEMBER 2021

www.sosinclasses.com +91 90000 36699 [email protected]

Pag

e48

Page 52: MONTHLY NEWS DIARY SEPTEMBER 2021

MONTHLY NEWS DIARY SEPTEMBER 2021

www.sosinclasses.com +91 90000 36699 [email protected]

Pag

e49

GS - 3

1. ECONOMY 1.1. ANALYSE THE ‘SHARP ECONOMIC RECOVERY’ OF 2021.

Context: Recently, the National Statistical Office data has revealed that the Indian economy grew at a record pace of 20.1% in April-June 2021, as compared to the corresponding period last year. Gross domestic product (GDP) contracted 24.4% in the same period, when the national lockdown due to the Covid-19 pandemic had nearly halted all economic activities.

Economic Recovery:

➢ The first quarter’s high growth rate has come despite a brutal second wave of the pandemic which peaked in April-May 2021.

➢ The sharp increases were largely due to the low base (-24.4%) of the first quarter of 2020-21.

➢ This growth reaffirms the government’s prediction of an imminent V-shaped recovery made last year.

➢ However, despite phenomenal economic recovery, the GDP in the first quarter is still 9.2% lower than the GDP in the same period during the pre-Covid year 2019-20.

➢ Amongst sectors, manufacturing and construction imparted a significant push to the economy in April-June, growing 49.63% and 68.3% respectively, over April-June 2020.

However, the services sector continued to lag. Sectors including ‘agriculture, forestry and fishing’ and ‘electricity, gas, water supply and other utility services’ are above the levels of the pre-Covid year of 2019-20.

Page 53: MONTHLY NEWS DIARY SEPTEMBER 2021

MONTHLY NEWS DIARY SEPTEMBER 2021

www.sosinclasses.com +91 90000 36699 [email protected]

Pag

e50

V-Shaped Economic Recovery:

A V-shaped recovery is characterized by a quick and sustained recovery in measures of economic performance after a sharp economic decline.

Such recoveries are generally spurred by a significant shift in economic activity caused by rapid readjustment of consumer demand and business investment spending.

About NSO: It is the central statistical agency of the Government mandated under the

Statistical Services Act 1980 under the Ministry of Statistics and Programme Implementation.

It is responsible for the development of arrangements for providing statistical information services to meet the needs of the Government and other users for information on which to base policy, planning, monitoring and management decisions.

Reports & Indices by NSO: ▪ Index of Industrial Production (IIP). ▪ Consumer Price Index (CPI). ▪ Sustainable Development Goals National Indicator Framework

Progress Report. ▪ Periodic Labour Force Survey (PLFS). ▪ GDP data.

Ways to measure total output of economy

Total Output in an economy can be measured by two ways:

❖ Measuring total demand: Gross Domestic Product (GDP)

❖ Measuring total supply: Gross Value Added (GVA)

GDP: It is the total monetary value of final goods and services, those that are bought by the final user and produced in a country in a given period of time. The GDP data show what is happening to the four engines of economic growth in any economy. These four engines are:

➢ Private Final Consumption Expenditure (C) ➢ Investment (I) ➢ Government Final Consumption Expenditure (G) ➢ Net Exports” (NX) (Exports-Imports) ➢ GDP = C + I + G + NX

GVA: It looks at how much value was added (in money terms) in different productive sectors (such as Agriculture, Electricity, etc.) of the economy. It tells which specific sectors are doing well and which are struggling to add value.

Page 54: MONTHLY NEWS DIARY SEPTEMBER 2021

MONTHLY NEWS DIARY SEPTEMBER 2021

www.sosinclasses.com +91 90000 36699 [email protected]

Pag

e51

Conclusion:

❖ The difference between these two absolute values will provide a sense of the role the government played. If the government earned more from taxes than what it spent on subsidies, GDP will be higher than GVA. If, on the other hand, the government provided subsidies more than its tax revenues, the absolute level of GVA would be higher than the absolute level of GDP.

1.2. WHAT IS ‘T+1 SETTLEMENT SYSTEM FOR SHARES’ AND ITS BENEFITS?

Background: Recently, Securities and Exchange Board of India (SEBI) allowed stock exchanges to start the T+1 system as an option in place of T+2 for completion of share transactions. It has been introduced on an optional basis in a move to enhance liquidity. SEBI is a statutory body established in 1992 in accordance with the provisions of the Securities and Exchange Board of India Act, 1992.

About:

If the stock exchange opts for the T+1 settlement cycle for a scrip, it will have to mandatorily continue with it for a minimum 6 months.

A scrip is a substitute or alternative to legal tender that entitles the bearer to receive something in return.

Thereafter, if it intends to switch back to T+2, it will do so by giving one month’s advance notice to the market. Any subsequent switch (from T+1 to T+2 or vice versa) will be subject to a minimum period.

T+1 vs T+2 Settlement:

✓ In T+2, if an investor sells shares, the settlement of the trade takes place in two working days (T+2) and the broker who handles the trade will get the money on the third day but will credit the amount in the investor’s account only by the fourth day. In effect, the investor will get the money only after three days.

✓ In T+1, settlement of the trade takes place in one working day and the investor will get the money on the following day.

✓ The move to T+1 will not require large operational or technical changes by market participants, nor will it cause fragmentation and risk to the core clearance and settlement ecosystem.

Benefits of T+1 Settlement:

➢ Reduced Settlement Time: A shortened cycle not only reduces settlement time but also reduces and frees up the capital required to collateralise that risk.

Settlement System In the securities industry, the trade settlement period refers to the time between the trade date that an order is executed in the market and the settlement date when a trade is considered final. On the last day of the settlement period, the buyer becomes the holder of record of the security.

Page 55: MONTHLY NEWS DIARY SEPTEMBER 2021

MONTHLY NEWS DIARY SEPTEMBER 2021

www.sosinclasses.com +91 90000 36699 [email protected]

Pag

e52

➢ Reduction in Unsettled Trade: It also reduces the number of outstanding unsettled trades at any instant, and thus decreases the unsettled exposure to Clearing Corporation by 50%.

➢ The narrower the settlement cycle, the narrower the time window for a counterparty insolvency/bankruptcy to impact the settlement of a trade.

➢ Reduction in Blocked Capital: Further, the capital blocked in the system to cover the risk of trades will get proportionately reduced with the number of outstanding unsettled trades at any point of time.

➢ Reduction in Systemic Risks: A shortened settlement cycle will help in reducing systemic risk.

Concerns of Foriegn Investors:

▪ Foreign investors have concerns about operational issues they would face while operating from different geographies - time zones, information flow process, and foreign exchange problems.

▪ They will also find it difficult to hedge their net India exposure in dollar terms at the end of the day under the T+1 system.

1.3. EXAMINE THE IRDAI (TRADE CREDIT INSURANCE) GUIDELINES OF 2021.

Background: Recently, the Insurance Regulatory and Development Authority of India (IRDAI) has issued revised guidelines for trade credit insurance.

Trade Credit Insurance:

➢ Trade credit insurance protects businesses against the risk of non-payment for goods and services.

➢ It usually covers a portfolio of buyers and indemnifies an agreed percentage of an invoice or invoices that remain unpaid as a result of protracted default or insolvency.

➢ It contributes to the economic growth of a country by facilitating trade and helps improve economic stability by addressing trade losses because of payment risks.

Coverage:

• It can be issued to sellers or suppliers of goods or services, factoring companies as defined in the Factoring Regulation Act, 2011 and banks and financial institutions.

• For banks and financial institutions and factoring companies, it covers the loss on account of non-receipt of payment from a buyer, due to commercial or political risks, against the bills and invoices purchased or discounted.

• Commercial risks include insolvency or extended default of the buyer, rejection by the buyer after delivery subject to conditions of contract, and rejection before shipment and non-receipt of payment on account of the collecting bank’s failure.

• Political risk cover is available only in case of buyers outside India and will include occurrence of war between the buyer’s country and India and also war, hostilities, civil war, rebellion, revolution, insurrection or other disturbances in the buyer’s country.

Page 56: MONTHLY NEWS DIARY SEPTEMBER 2021

MONTHLY NEWS DIARY SEPTEMBER 2021

www.sosinclasses.com +91 90000 36699 [email protected]

Pag

e53

Applicability: These guidelines will apply to all insurers transacting general insurance business, registered under the Insurance Act, 1938. However, ECGC Ltd (formerly Export Credit Guarantee Corporation of India Ltd) is exempted from the application of these guidelines.

Benefits of the Move:

▪ It will facilitate general insurance companies to help businesses manage country risk, open up access to new markets and manage non-payment risk associated with trade financing portfolios.

▪ It will also enable general insurance companies to offer trade credit insurance with customised covers to improve businesses for the Micro, Small and Medium Enterprises (MSMEs), considering the evolving insurance risk needs of these enterprises.

1.4. CRITICALLY ANALYSE THE INSOLVENCY RESOLUTION PROCESS IN INDIA.

Background: Recently, the Supreme Court (SC) held that the Committee of Creditors (CoC) approved resolution plan under the Insolvency and Bankruptcy Code (IBC) submitted to the National Company Law Tribunal (NCLT) cannot be modified. Earlier, in July 2021 the government introduced the Insolvency and Bankruptcy Code (Amendment Bill), 2021 in the Lok Sabha.

SC’s Ruling:

➢ No Modifications: The adjudicating authority cannot allow modifications or withdrawals of Resolution Plans approved by the Committee of Creditors at the behest of the successful Resolution Applicant once the plan is submitted to it.

➢ Timely Completion: Corporate Insolvency Resolution Process (CIRP) carried out under the IBC must be completed within 330 days as laid down by the Code. It quoted a report of the Parliamentary standing committee on finance which stated that 71% cases are pending for more than 180 days before NCLT (National Company Law Tribunal). And asked the NCLT and the National Company Law Appellate Tribunal (NCLAT) to stick to this, and should therefore, while deciding IBC matters, respect the deadline keeping in mind the effect of such delays on the insolvency resolution process.

➢ Judicial delay was one of the major reasons for the failure of the insolvency regime that was in effect prior to the IBC.

➢ The time limit can be extended only in exceptional circumstances as otherwise, the open-ended process for further negotiations or a withdrawal, would have a deleterious impact on the Corporate Debtor, its creditors, and the economy at large as the liquidation value depletes with the passage of time.

Insolvency Resolution Process in India:

• Eligibility: Under IBC, companies (both private and public limited company) and Limited Liability Partnerships (LLP) can be considered as defaulting corporate debtors. A corporate debtor is any corporate organization which owes a debt to any person.

Page 57: MONTHLY NEWS DIARY SEPTEMBER 2021

MONTHLY NEWS DIARY SEPTEMBER 2021

www.sosinclasses.com +91 90000 36699 [email protected]

Pag

e54

• Default Amount: The IBC can be triggered if there is a minimum default of Rs 1 Crore. This process can be triggered by way of filing an application before the NCLT.

• Resolution Initiation: The process can be initiated by two classes of creditors which would include financial creditors and operational creditors.

• Creditors: A Creditor means any person to whom a debt is owed and includes a financial creditor, an operational creditor, etc.

• Financial Creditors: The financial creditor in simple terms is the institution that provided money to the corporate entity in the form of loans, bonds etc. E.g. banks.

• Operational Creditors: An operational creditor is the entity who has a claim for providing any of the four categories to the defaulted corporate- goods, services, employment, and Government dues (central govt, state or local bodies).

• Appointment of Interim Resolution Professional: As soon as the matter is admitted by the NCLT, the NCLT proceeds with the appointment of an Interim Resolution Professional (IRP) who takes over the management of the defaulting debtor.

• Committee of Creditors (CoC): A committee consisting only of the financial creditors i.e. the CoC is formed by the IRP. Only operational creditors having aggregate dues of at least 10% of the total debt are invited into the meeting of CoC (Operational creditors are not a member of CoC). The operational creditors don’t have any voting power.

• Corporate Insolvency Resolution Process (CIRP): It includes necessary steps to revive the company such as raising fresh funds for operation, looking for a new buyer to sell the company as a going concern, etc. The CoC takes a decision regarding the future of the outstanding debt owed to it. The resolution plan can be implemented only if it has been approved by 66% of the creditors in the CoC. The IBC (Amendment Bill), 2021 introduced an alternate insolvency resolution process for Micro, Small and Medium Enterprises (MSMEs) with defaults up to Rs 1 crore called the Pre-packaged Insolvency Resolution Process (PIRP).

• Liquidation Proceedings: In the event a resolution plan is not submitted or not approved by the Committee of Creditors (CoC), the CIRP process is deemed to have failed. In such a situation the liquidation proceedings commence subject to the order of the tribunal.

Way Forward

Some of the issues faced in the implementation of IBC can be eased by conducting timely colloquium for judges of NCLT and increasing interaction between practitioners from various jurisdictions. NCLTs according to higher priority to applications filed in respect of avoidable transactions such as extortionate, preferential, undervalued, and fraudulent transactions. NCLTs discourage frequent adjournments and ensure timeliness of the resolution process. Focus should be on sensitizing various government and statutory authorities about the treatment of government and statutory dues under IBC to reduce the scope for litigation and consequent delay in the resolution of companies under corporate insolvency.

Page 58: MONTHLY NEWS DIARY SEPTEMBER 2021

MONTHLY NEWS DIARY SEPTEMBER 2021

www.sosinclasses.com +91 90000 36699 [email protected]

Pag

e55

1.5. WHAT IS ‘NEW BAD BANK STRUCTURE’ AND ITS SIGNIFICANCE?

Background: Recently, the Union Cabinet approved the Rs 30,600 crore guarantee to back Security Receipts issued by National Asset Reconstruction Company Limited (NARCL) for acquiring stressed loan assets. The NARCL is a part of a new Bad bank structure that was announced in the Budget 2021.

The New Bad Bank Structure:

➢ For resolution of huge NPAs (Non-Performing Assets) in the Indian Banking sector, the government of India has set up two new entities to acquire stressed assets from banks and then sell them in the market.

➢ NPA refers to a classification for loans or advances that are in default or in arrears.

NARCL: NARCL has been incorporated under the Companies Act and has applied to the Reserve Bank of India for a license as an Asset Reconstruction Company (ARC). NARCL will acquire stressed assets worth about Rs 2 lakh crore from various commercial banks in different phases. Public Sector Banks (PSBs) will maintain 51% ownership in NARCL.

IDRCL: Another entity, India Debt Resolution Company Ltd (IDRCL), will then try to sell the stressed assets in the market. PSBs and Public Financial Institutes (FIs) will hold a maximum of 49% stake in IDRCL. The remaining 51% stake will be with private-sector lenders. The NARCL-IDRCL structure is the new bad bank structure.

Need for NARCL-IDRCL Structure:

▪ Existing ARCs have been helpful in the resolution of stressed assets, especially for smaller value loans.

▪ Various available resolution mechanisms, including Insolvency and Bankruptcy Code (IBC), have proved to be useful.

▪ However, considering the large stock of legacy NPAs, additional options/alternatives are needed and thus, the NARCL-IRDCL structure was announced in the Union Budget 2021.

Working of NARCL-IDRCL and Guarantee Offered: The NARCL will first purchase bad loans from banks. It will pay 15% of the agreed price in cash and the remaining 85% will be in the form of “Security Receipts”. When the assets are sold, with the help of IDRCL, the commercial banks will be paid back the rest. If the bad bank is unable to sell the bad loan, or has to sell it at a loss, then the government guarantee will be invoked. The difference between what the commercial bank was supposed to get and what the bad bank was able to raise will be paid from the Rs 30,600 crore that has been provided by the government. This guarantee is extended for a period of five years.

Note Security receipts are defined under

section 2(1) of SARFAESI Act. It means a receipt or other security,

issued by an asset reconstruction company to any qualified buyer pursuant to a scheme, evidencing the purchase or acquisition by the holder, thereof, of an undivided right, title or interest in the financial asset involved in securitization.

Page 59: MONTHLY NEWS DIARY SEPTEMBER 2021

MONTHLY NEWS DIARY SEPTEMBER 2021

www.sosinclasses.com +91 90000 36699 [email protected]

Pag

e56

About Bad Bank

• The bad bank is an Asset Reconstruction Company (ARC) or an Asset Management Company (AMC) that takes over the bad loans of commercial banks, manages them and finally recovers the money over a period.

• The bad bank is not involved in lending and taking deposits, but helps commercial banks clean up their balance sheets and resolve bad loans.

• The takeover of bad loans is normally below the book value of the loan and the bad bank tries to recover as much as possible subsequently.

Effect of Bad Bank:

➢ Commercial Banks’ Perspective: Commercial banks are saddled with high NPA levels, setting up of the Bad bank will help. That’s because such a bank will get rid of all its toxic assets, which were reducing its profits, in one quick move. When the recovery money is paid back, it will further improve the bank’s position. Meanwhile, it can start lending again.

➢ Government and Taxpayer Perspective: Whether it is recapitalizing PSBs laden with bad loans or giving guarantees for security receipts, the money is coming from the taxpayers’ pocket. While recapitalization and such guarantees are often designated as “reforms”, they are band aids at best. The only sustainable solution is to improve the lending operation in PSBs. The plan of bailing out commercial banks will collapse if the bad bank is unable to sell such impaired assets in the market. The burden indeed will fall upon the taxpayer.

Way Forward

So long as Public Sector Banks’ managements remain beholden to politicians and bureaucrats, their deficit in professionalism will remain and subsequently, prudential norms in lending will continue to suffer. Therefore, a bad bank is a good idea, but the main challenge lies with tackling the underlying structural problems in the banking system and announcing reforms accordingly.

1.6. ANALYSE THE PLI SCHEME FOR AUTO & DRONE SECTOR.

Background: Recently, the Union Cabinet approved a Rs. 26,058 crore Production Linked Incentive (PLI) scheme for auto, auto-components, and Drone industries to enhance India’s manufacturing capabilities. The scheme for the sector is part of the overall production-linked incentives announced for 13 sectors in the Union Budget 2021-22 with an outlay of Rs 1.97 lakh crore. It is a significant milestone in India’s journey towards ‘Atma Nirbharta’ and will enable the country to join the top order of Auto and Drone manufacturing nations.

Page 60: MONTHLY NEWS DIARY SEPTEMBER 2021

MONTHLY NEWS DIARY SEPTEMBER 2021

www.sosinclasses.com +91 90000 36699 [email protected]

Pag

e57

PLI Scheme:

➢ PLI scheme, introduced in March 2020, aims to give companies incentives on incremental sales from products manufactured in domestic units.

➢ Apart from inviting foreign companies to set shop in India, it also aims to encourage local companies to set up or expand existing manufacturing units.

➢ It has also been approved for sectors such as automobiles, pharmaceuticals, IT hardware including laptops, mobile phones & telecom equipment, white goods, chemical cells, food processing, Textile Sector etc.

PLI for Auto Sector:

▪ It excludes conventional petrol, diesel and CNG segments (Internal Combustion Engine) since it has sufficient capacity in India.

▪ It is incentivizing only advanced automotive technologies or auto components whose supply chains are weak, dormant, or non-existing.

▪ It is aimed at boosting new technology and the economy of clean fuels.

Components:

➢ Champion OEM (Original Equipment Manufacturers) Scheme: It is a sales value linked plan, applicable to Battery Electric and Hydrogen Fuel Cell Vehicles of all segments.

Page 61: MONTHLY NEWS DIARY SEPTEMBER 2021

MONTHLY NEWS DIARY SEPTEMBER 2021

www.sosinclasses.com +91 90000 36699 [email protected]

Pag

e58

➢ Champion Incentive Scheme: It is a Sales Value Linked plan for advanced technology components, Complete and Semi-Knocked Down (CKD/SKD) kits, vehicle aggregates of 2-wheelers, 3-wheelers, passenger vehicles, commercial vehicles and tractors.

Significance:

▪ This scheme along with the already launched PLI for Advanced Chemistry Cell and Faster Adoption of Manufacturing of Electric Vehicles (FAME) Scheme will give a big boost to the manufacture of Electric Vehicles.

▪ It will contribute towards reducing carbon emissions and oil imports. ▪ It will encourage production of auto components using advanced technologies that will

boost localisation, domestic manufacturing and also attract foreign investments. ▪ It will help setting up new facilities and create more jobs. It is expected to generate 7.5

lakh jobs for the auto sector.

PLI for Drone Sector:

It covers a wide variety of drone components, including airframe, propulsion systems, power systems, batteries, inertial measurement unit, flight control module, ground control station, communication systems, cameras, sensors, spraying systems, emergency recovery system, and trackers.

It is expected to bring fresh investments of over Rs 5,000 crore and incremental production of over Rs 1,500 crore and create additional employment of about 10,000 jobs.

Significance: It will encourage entrepreneurs to strive towards building drones, components, and software for the global market. It will also open many more verticals for the utilization of drones. It will help reduce imports. At present 90 %of the drones in India are imported. The government intends to make India into a global drone hub by 2030.

1.7 WHAT IS SUFFICIENCY ECONOMY PHILOSOPHY OF THAILAND ?

Background: Thailand believes that its homegrown development approach of Sufficiency Economy Philosophy (SEP) can serve as an alternative approach to achieving the Sustainable Development Goals (SDGs). In 2020, Indian PM announced Atmanirbhar Bharat or Self-reliant India Movement, which has a similar approach to make India and its citizens independent and self-reliant in all senses. When India speaks of self-

Sufficiency Economy Philosophy: It is an innovative approach to development

designed for practical application over a wide range of problems and situations.

It is also part of the fundamental administration policy of the state (Thailand).

It was introduced in Thailand after the Asian financial crisis in 1997.

It is a philosophy that guides one’s inner thinking to immunize oneself from external shocks and can be applied in any setting and at any levels.

Page 62: MONTHLY NEWS DIARY SEPTEMBER 2021

MONTHLY NEWS DIARY SEPTEMBER 2021

www.sosinclasses.com +91 90000 36699 [email protected]

Pag

e59

reliance, it does not advocate for a self-centered system, there is also a concern for the whole world’s happiness, cooperation and peace.

Levels:

Individual and Family Level: It means living a simple life, living within one's means, and refraining from taking advantage of other people.

Community Level: It involves joining together to participate in decision-making, developing mutually beneficial knowledge, and appropriately applying technology.

National Level: It postulates a holistic approach with an emphasis upon appropriateness, competitive advantage, low risk, and avoiding over-investment. It involves keeping abreast with what is happening elsewhere in the world, hedging investments, and reducing imports and over dependence on other countries.

Pillars:

▪ Knowledge: It enables effective planning and execution of developmental activities.

▪ Ethics and Values: It enhances human development by emphasizing honesty, altruism, and perseverance, with creating active, engaged citizens, and promoting good governance as the ultimate goal.

Principles:

▪ Moderation: It entails producing and consuming within one’s capacity and avoiding overindulgence.

▪ Reasonableness: It is using one’s mental faculties to examine the causes and consequences of actions on one’s well-being, household and community.

▪ Prudence: It refers to risk management in order to be prepared for impacts from any disruptions.

Page 63: MONTHLY NEWS DIARY SEPTEMBER 2021

MONTHLY NEWS DIARY SEPTEMBER 2021

www.sosinclasses.com +91 90000 36699 [email protected]

Pag

e60

1.8 World Bank Stops ‘Ease of Doing Business’ Report. Explain.

Background: The World Bank would discontinue the practice of issuing ‘Doing Business report’ following an investigation reported “data irregularities” in its 2018 and 2020 editions (released in 2017 and 2019, respectively) and possible “ethical matters” involving bank staff.

It will be working on a new approach to assessing the business and investment climate.

Ease of Doing Business Report:

➢ The report was introduced in 2003 to provide an assessment of objective measures of business regulations and their enforcement across 190 economies on ten parameters affecting a business through its life cycle.

➢ 10 different parameters namely, Starting a Business, Dealing with Construction permits, Electricity availability, Property registration, Credit availability, Protecting minority Investors, Paying Taxes, Trading across borders, Contracts enforcement, and Resolving Insolvency.

➢ It ranks countries on the basis of Distance to Frontier (DTF) score that highlights the gap of an economy with respect to the global best practice. For example, a score of 75 means an economy was 25 percentage points away from the frontier constructed from the best performances across all economies and across time.

India’s Performance:

➢ Notably, in three reports, released in 2017, 2018 and 2019, India ranked among the top 10 economies showing “the most notable improvement”.

➢ Of the 79 positions in the Bank’s Doing Business rankings that India gained between 2014 and 2019, 67 rank improvements happened 2017 onwards, with the biggest 30-rank jump happening in the Doing Business 2018 report, released in October 2017.

➢ The latest report, published in October 2019, placed India at 63rd in Doing Business, compared with 77th in 2018 and 100 in 2017.

➢ India, along with other top improvers, had implemented 59 regulatory reforms in 2018-19, accounting for a fifth of all reforms recorded worldwide.

➢ During 2018-19, India had implemented reforms across parameters such as ‘starting a business’, ‘dealing with construction permits’, ‘trading across borders’, and ‘resolving insolvency’. The government’s goal was to be among the top 50 economies by 2020.

➢ The scores for India used to be based on coverage of just two cities, with Mumbai carrying a weight of 47% and Delhi a weight of 53%.

Page 64: MONTHLY NEWS DIARY SEPTEMBER 2021

MONTHLY NEWS DIARY SEPTEMBER 2021

www.sosinclasses.com +91 90000 36699 [email protected]

Pag

e61

World Bank It was created in 1944, as the International Bank for Reconstruction and

Development (IBRD) along with the International Monetary Fund (IMF). The IBRD later became the World Bank.

The World Bank Group is a unique global partnership of five institutions working for sustainable solutions that reduce poverty and build shared prosperity in developing countries.

It has 189 member countries. India is also a member country. Major reports:

Ease of Doing Business. Human Capital Index. World Development Report.

Page 65: MONTHLY NEWS DIARY SEPTEMBER 2021

MONTHLY NEWS DIARY SEPTEMBER 2021

www.sosinclasses.com +91 90000 36699 [email protected]

Pag

e62

2. ENVIRONMENT 2.1. ASSESS THE CHALLENGES IN MANAGING NATURAL RESOURCES.

Context: A project in Meghalaya empowers communities to take informed action pertaining to their environment.

Depletion of natural resources in Meghalaya

• In recent years, many parts of Meghalaya have witnessed the loss of forest cover and natural resources have rapidly deteriorated.

• The State, known to have spots designated as the ‘wettest places’ on earth, is now facing a severe water crisis.

• Natural resource management becomes critical in this context.

Providing knowledge: Landscape Management Project

➢ The government wanted to see if, when provided with the correct knowledge, solutions to problems can be devised and even implemented by community members themselves.

➢ The World Bank-supported Meghalaya Community-Led Landscape Management Project seeks to reactivating the community’s connection to natural resources and enabling them to tackle the resource crisis.

➢ How the project worked: cross-functional teams with diverse expertise were set up.

➢ The Mahatma Gandhi National Rural Employment Guarantee Scheme became the main scheme channelizing resources to impact poor households so that there was systematic convergence of all line departments such as agriculture, horticulture, soil, and water conservation.

➢ The programme leverages technology and the youth population.

➢ Leveraging technology, more than 2,000 village community facilitators have already been trained and are working towards climate change reversal.

➢ To build autonomy, simple tools are used.

➢ They have been designed keeping in mind many things: creating community agency, building the capacities of all persons in the programme, and ensuring frequent interactions among them.

➢ Technology empowers them with real-time data, which in turn results in better programme governance, transparency, and accountability.

Challenges in natural resource management: Traditional practices on sustainable use

of natural resources have been passed down from one generation to another.

This indigenous knowledge began to slowly fade, however, owing to population growth, the quest for unsustainable developmental activities, and indiscriminate exploitation of natural resources.

Another roadblock to natural resource management was knowledge inaccessibility among rural communities.

Page 66: MONTHLY NEWS DIARY SEPTEMBER 2021

MONTHLY NEWS DIARY SEPTEMBER 2021

www.sosinclasses.com +91 90000 36699 [email protected]

Pag

e63

➢ Communities are now able to articulate the complexities of their problems through a scientific lens and create their own natural resource management plans.

➢ To carry forward this momentum, there is a plan to launch a Centre of Excellence in Meghalaya, a one-stop centre for natural resources management.

Conclusion:

❖ The project intends to empower thousands of village community facilitators and enable them to articulate the complexities of their problems through a scientific lens and create their own natural resource management plans.

2.2. EXPLAIN THE SIGNIFICANCE OF THE MUMBAI CLIMATE ACTION PLAN (MCAP).

The Brihanmumbai Municipal Corporation (BMC) is drafting a Mumbai Climate Action Plan (MCAP) in a bid to tackle climate challenges.

What is the Mumbai Climate Action Plan?

• Amid warnings of climate change leading to extreme weather events in the city, the civic body has started preparing the Mumbai Climate Action Plan (MCAP).

• It will look at climate resilience with mitigation and adaptation strategies by focusing on six areas —

1. Sustainable waste management

2. Urban greening and Biodiversity

3. Urban flooding and Water Resource Management

4. Building Energy Efficiency

5. Air Quality and

6. Sustainable Mobility

• The plan is expected to be ready by November ahead of the United Nations Climate Change (COP26) conference at Glasgow, Scotland.

Why does Mumbai need a climate action plan?

Mumbai’s climate action plan will help set a vision and implement strategies to fight these climate challenges with mitigation and adaptation steps.

• Flash floods: As per a study conducted by the World Resource Institute (WRI) India, the city will face two major climate challenges — the rise in temperature, and extreme rain events which will lead to flooding.

• Temperature rise: The city has seen a constant rise in temperature after 2007, and a substantial increase in intense rainfall and storm events in the last five years.

Page 67: MONTHLY NEWS DIARY SEPTEMBER 2021

MONTHLY NEWS DIARY SEPTEMBER 2021

www.sosinclasses.com +91 90000 36699 [email protected]

Pag

e64

• Sea level rise: A recent report from the IPCC has warned that at least 12 Indian coastal cities including Mumbai will face a sea rise of 0.1 metres to 0.3 metres in the next three decades due to climate change.

What is the greenhouse gas emission of the city?

• The data show that Mumbai’s greenhouse gas emission was 34.3 million tonnes in 2019, and of which 24.23 million tonnes or 71 per cent came from the energy sector which is mainly based on coal.

• At least 24 per cent or 82,21,902 tonnes is from transport, and the remaining 5 per cent or 18,53,741 tonnes from solid waste management.

• The maximum contribution from the energy sector was mainly due to domestic and commercial usage of electricity.

As per the data, 95 percent of Mumbai’s electricity is coal-based and needs to be shifted to renewable energy to bring down emissions.

2.3. EXPLAIN THE INITIATIVES TAKEN UNDER THE NITI AAYOG’S METHANOL ECONOMY PROGRAMME.

Background: Recently, the first Indigenously Designed High Ash Coal Gasification Based Methanol Production Plant has been opened in Hyderabad. With this, Government owned engineering firm BHEL (Bharat Heavy Electricals Limited) has successfully demonstrated a facility to create methanol from high ash Indian coal.

About Methanol: Methanol is utilized as a motor fuel, to power ship engines, and to generate clean

power all over the world. However, the majority of worldwide methanol production is derived from natural gas, which is a relatively easy process.

Since India doesn’t have much of the natural gas reserves, producing methanol from imported natural gas leads to outflow of foreign exchange and is uneconomical because of higher prices.

The next best option is to utilize India's abundant coal. However, due to the high ash percentage of Indian coal, most internationally accessible technology will not be adequate.

To address this issue, BHEL successfully demonstrated a facility to create 0.25 TPD (Ton per Day) Methanol from high ash Indian coal using a 1.2 TPD Fluidized bed gasifier.

The methanol purity of the crude methanol produced is between 98 and 99.5%. This is part of NITI Aayog's 'Methanol Economy' programme that is aimed at reducing

India's oil import bill, greenhouse gas (GHG) emissions, and converting coal reserves and municipal solid waste into methanol.

Also, this in-house capability will assist India's coal gasification mission and coal-to-hydrogen production for Hydrogen Mission.

Page 68: MONTHLY NEWS DIARY SEPTEMBER 2021

MONTHLY NEWS DIARY SEPTEMBER 2021

www.sosinclasses.com +91 90000 36699 [email protected]

Pag

e65

NITI Aayog’s Methanol Economy Programme:

➢ Significance of Methanol: Methanol is a low carbon, hydrogen carrier fuel produced from high ash coal, agricultural residue, CO2 from thermal power plants and natural gas. It is the best pathway for meeting India’s commitment to COP 21 (Paris Agreement).

➢ Methanol vis-a-vis-Petrol and Diesel: Although slightly lower in energy content than petrol and diesel, methanol can replace both these fuels in the transport sector (road, rail and marine), energy sector (comprising boilers, process heating modules, tractors and commercial vehicles) and retail cooking (replacing LPG [partially], kerosene and wood charcoal).

➢ Environmental and Economic Impact: Blending of 15% methanol in gasoline can result in at least 15% reduction in the import of gasoline/crude oil. In addition, this would bring down GHG emissions by 20% in terms of particulate matter, NOx, and SOx, thereby improving the urban air quality. The Methanol Economy will also create close to 5 million jobs through methanol production/application and distribution services. Additionally, Rs 6000 crore can be saved annually by blending 20% DME (Di-methyl Ether, a derivative of methanol) in LPG. This will help the consumer in saving between Rs 50-100 per cylinder.

Initiatives Taken:

• The Bureau of Indian Standards has notified 20% DME blending with LPG, and a notification for M-15, M-85, M-100 blends has been issued by the Ministry of Road, Transport and Highways.

• In October 2018, Assam Petrochemicals launched Asia’s first canister-based methanol cooking fuel programme. This initiative is in line with the Prime Minister’s vision of striving towards the provision of a clean, cost-effective, and pollution-free cooking medium.

• Five methanol plants based on high ash coal, five DME plants, and one natural gas-based methanol production plant with a capacity of 20 MMT/annum, in a joint venture with Israel, have been planned to be set up.

• Three boats and seven cargo vessels are being built by the Cochin Shipyard Limited for Inland Waterways Authority of India to use methanol as a marine fuel.

Way Forward: India, with 125 billion Tonnes of proven Coal reserves and 500 million tonnes of Biomass generated every year has a huge potential for ensuring energy security based on alternate feedstock and fuels. However, Methanol does not get the same attention from the government as EV (Electric Vehicles), even though the former can come in faster. There is significant work needed to implement the Methanol Economy holistically. The development of methanol-based technology can turn energy-importing India into an energy exporting country.

Page 69: MONTHLY NEWS DIARY SEPTEMBER 2021

MONTHLY NEWS DIARY SEPTEMBER 2021

www.sosinclasses.com +91 90000 36699 [email protected]

Pag

e66

2.4. ASSESS THE FACTORS LEADING TO THE HANGING RAINFALL PATTERN IN NORTHEAST INDIA.

Context: Recently, an analysis showed a changing rainfall pattern in Northeast (NE) India because of Climate Change. The National Action Plan on Climate Change (NAPCC) was launched in 2008 by the Prime Minister's Council on Climate Change. It identifies measures that promote India's development objectives while also yielding co-benefits for addressing climate change effectively.

North-East rainfall:

• NE normally receives heavy rainfall during the Monsoon months (June-September), but has changed its course in recent years. The rains come in quick bursts and Flood the region, followed by elongated dry periods that border on drought.

• A research paper published in 2018 found that the monsoon rainfall in NE decreased by 355 mm between 1979 and 2014. Out of this, 30-50 mm decrease was due to a reduction in local Moisture levels.

• Because of its unique topology and steep slopes suddenly giving way to plains, the region is prone to river course changing.

• NE is mostly hilly and is an extension of the Indo-Gangetic Plains, the region is highly sensitive to changes in regional and global climate. Pre-monsoon and monsoon are the rainy seasons of northeast India.

• In most of the NE states the rainfall during monsoon has declined in two decades below the Long Period Average (LPA). The number of rainy days decreased in most districts to the north of the Brahmaputra. This means that increased rainfall now happens over fewer days, increasing chances of river flooding.

Page 70: MONTHLY NEWS DIARY SEPTEMBER 2021

MONTHLY NEWS DIARY SEPTEMBER 2021

www.sosinclasses.com +91 90000 36699 [email protected]

Pag

e67

Factors Causing Rainfall Patterns Change:

➢ Moisture and Drought Together: An aspect of warming that influences rainfall is drying of the land, which increases the frequency and intensity of dry periods and Droughts. Increase in moisture and the drying up together change the rainfall patterns in unpredictable ways.

➢ Increased Snowfall in the Eurasian Region: Increased snowfall in the Eurasian region also impacts monsoon rainfall in NE India as the excessive snowfall in Eurasia causes cooling of the atmosphere of the region, which triggers events eventually leading to a weak summer monsoon season there.

➢ Change in Pacific Decadal Oscillation (PDO): Sea surface temperatures over the subtropical Pacific Ocean, which vary in a cycle and each phase of which lasts a decade. The peak comes every 20 years and is known as the Pacific Decadal Oscillation (PDO). It may have an impact on the monsoon rainfall in NE. PDO is also being influenced by global warming as it decreases the difference of temperatures among the layers of the ocean.

➢ Sunspot Epoch: During the monsoon, rainfall patterns in NE differed significantly from one Sunspot epoch to another, suggesting differential intensification of the seasonal trough of low pressure over the country. Sunspot Epochs are alternating periods of increased and decreased activity on the Sun’s surface that influence the climate of Earth.

Impact: The changing rainfall pattern, especially during the monsoon season, affects the flow of rivers, extent of snow cover and health of mountain springs, which in turn have an impact on livelihoods, especially agriculture and fishing, forest flora growth, animal, and bird habitat (and behaviour) and other ecosystem aspects. There is some evidence of rivers such as Subansiri, Dibang (tributaries of Brahmaputra) and Brahmaputra changing courses in unexpected ways. Extreme rainfall events being caused by global warming trigger a cascade of events such as accelerated Soil Erosion along the hill slopes devoid of forest cover. This increases surface run-off of rivers and change their course.

2.5. What is the Impact of Global warming on Permafrost?

Background: According to the latest IPCC report, increasing global warming will result in reductions in Arctic permafrost and the thawing of the ground is expected to release greenhouse gases like methane and carbon dioxide.

Page 71: MONTHLY NEWS DIARY SEPTEMBER 2021

MONTHLY NEWS DIARY SEPTEMBER 2021

www.sosinclasses.com +91 90000 36699 [email protected]

Pag

e68

Permafrost:

• Permafrost is any ground that remains completely frozen - 32°F (0°C) or colder - for at least two years straight.

• These permanently frozen grounds are most common in regions with high mountains and in Earth’s higher latitudes - near the North and South Poles. Permafrost covers about 15% of the land area of the globe.

• Although the ground is frozen, permafrost regions are not always covered in snow.

• Landscapes with large stretches of permafrost are often called tundra. The word tundra is a Finnish word referring to a treeless plain. Tundra is found at high latitudes and at high altitudes, where the permafrost has a very thin active layer.

Concerns Regarding Melting Permafrost:

➢ Affects Infrastructure: It will affect countries where roads or buildings were constructed on permafrost.

➢ Release of Greenhouse Gases: It entombed and froze the organic material in the ground. If the ground begins to thaw, this material will become available for microorganisms to break down. In some environments, the microorganisms will release carbon dioxide, and in others release methane which is about 25 to 30 times more potent as a greenhouse gas than carbon dioxide.

➢ Change from Carbon Storehouse to Carbon Emitters: Some permafrost regions have changed from being a carbon storehouse to being places that are net emitters of carbon.

➢ Increase in the Number of Forest Fires: This year Russia witnessed a forest fire whose total area was the size of Portugal. Usually, after a fire, one expects the forest to grow back in the next 50 years to 60 years. This restores the carbon stock in the ecosystem. But in the tundra, the peat is where the organic material is and this takes a very long

Page 72: MONTHLY NEWS DIARY SEPTEMBER 2021

MONTHLY NEWS DIARY SEPTEMBER 2021

www.sosinclasses.com +91 90000 36699 [email protected]

Pag

e69

time to accumulate. So if peat is burned and released into the atmosphere, then it will take centuries to restore that carbon stock at ground level.

➢ Releasing New Bacteria or Viruses: The environment now is so much more suitable than during the Ice Age for not just human life, but also the evolution or development of viruses and bacteria. So, the chances of emerging new bacteria or viruses cannot be ignored.

Steps to be Taken:

✓ Stop Rapid Climate Change: In order to curtail climate change and save the permafrost, it is indispensable that global CO2 emissions be reduced by 45% over the next decade, and that they fall to zero after 2050.

✓ Slow Down Erosion: The scientific journal Nature suggested building a 100-metre-long dam in front of the Jakobshavn glacier (Greenland), the worst affected by Arctic melting, to contain its erosion.

✓ Combine Artificial Icebergs: Indonesian architect has won an award for his project Refreeze the Arctic, which consists of collecting water from melted glaciers, desalinating it and refreezing it to create large hexagonal ice blocks.

✓ Increase Their Thickness: Some researchers propose a solution to manufacture more ice. Their proposal consists of collecting ice from below the glacier through pumps driven by wind power to spread it over the upper ice caps, so that it will freeze, thus strengthening the consistency.

✓ People’s Awareness: The tundra and the permafrost beneath it may seem far away, but no matter where we live, the everyday choices we make contribute to climate change.

By reducing our carbon footprint, investing in energy-efficient products, and supporting climate-friendly businesses, legislation, and policies, we can help preserve the world’s permafrost and avert a vicious cycle of an ever-warming planet.

2.6. EXAMINE THE CHALLENGES IN THE PRODUCTION OF ‘COAL-BASED HYDROGEN’.

Background: Recently, the Union Government constituted a Task force and Expert committee to prepare a road map for coal-based hydrogen production (Black Hydrogen). The Task Force is also responsible for coordination with the Coal Gasification Mission and NITI Aayog.

Coal-Based Hydrogen Production:

➢ Coal (one of the Hydrocarbon Fuels) is one of the important sources of hydrogen making apart from natural gas and renewable energy through Electrolysis.

➢ However, Coal has not been encouraged in hydrogen production because of the fear of Carbon Emission while extracting hydrogen via coal.

➢ Almost 100% of hydrogen produced in India is through natural gas (Grey Hydrogen).

Benefit: Cost of hydrogen produced from coal can be cheaper and less sensitive to imports.

Challenge: Production of hydrogen from coal will have challenges in terms of high emissions and CCUS (Carbon capture, utilization and storage) will play an important role. Carbon monoxide and carbon dioxide formed during the coal to hydrogen process have to be trapped and stored in an environmentally sustainable manner (CCS and CCUS).

Page 73: MONTHLY NEWS DIARY SEPTEMBER 2021

MONTHLY NEWS DIARY SEPTEMBER 2021

www.sosinclasses.com +91 90000 36699 [email protected]

Pag

e70

Hydrogen Economy:

• It is an economy that relies on hydrogen as the commercial fuel that would deliver a substantial fraction of a nation’s energy and services.

• Hydrogen is a zero-carbon fuel and is considered an alternative to fuel and a key source of clean energy. It can be produced from renewable sources of energy such as solar and wind.

• It is an envisioned future where hydrogen is used as fuel for vehicles, energy storage and long-distance transport of energy. The different pathways to use hydrogen economy includes hydrogen production, storage, transport, and utilization.

• In 1970, the term 'Hydrogen Economy' was coined by John Bockris. He mentioned that a hydrogen economy can replace the current hydrocarbon-based economy, leading to a cleaner environment.

Present Scenario:

The current global demand for hydrogen is 70 million metric tons, most of which is being produced from fossil fuels- 76% from natural gas and 23% from coal and remaining from the electrolysis of water-- consumes 6% of the global natural gas and 2% of the global coal.

This results in CO2 emissions of around 830Mt/year out of which only 130Mt/year is being captured and used in the fertilizer industry.

Currently, much of the hydrogen produced is used for oil refining (33%), ammonia (27%), methanol production (11%), steel production (3%) and others.

Related Initiatives

▪ National Hydrogen Energy Mission.

▪ Hydrogen Fuel Cell Based Vehicles.

▪ Green Hydrogen Mobility project.

Page 74: MONTHLY NEWS DIARY SEPTEMBER 2021

MONTHLY NEWS DIARY SEPTEMBER 2021

www.sosinclasses.com +91 90000 36699 [email protected]

Pag

e71

2.7. EXAMINE THE ARSENIC CONTAMINATION OF FOOD CHAIN.

Context: A recent study in Bihar has found Arsenic contamination not only in groundwater but in the food chain as well. The research study was a part of the Project Nature and Nurture in Arsenic Induced Toxicity of Bihar jointly funded by the British Council in the United Kingdom and Department of Science and Technology in India.

Major Findings:

➢ Food Chain Contamination: Arsenic has found its way into the food chain - mainly rice, wheat and potato. Arsenic contamination in groundwater has been a growing concern in several parts of the country. Arsenic is present in the groundwater as it is used on a large scale for irrigation by farmers. That is how it finds its way into the food chain as well.

➢ Food vs Water Contamination: The food had more arsenic content than drinking water, even when arsenic levels in drinking water was above the World Health Organization (WHO) provisional guide value of 10 micrograms per litre (μg/L). The concentration was higher in cooked rice compared to raw rice.

Arsenic:

It is an odourless and tasteless metalloid widely distributed in the earth’s crust. It is naturally present at high levels in the earth crust and groundwater of a number of countries. It is highly toxic in its inorganic form.

Arsenic Poisoning:

▪ It can get into the human body through drinking water as well as eating food that has been contaminated with arsenic.

Page 75: MONTHLY NEWS DIARY SEPTEMBER 2021

MONTHLY NEWS DIARY SEPTEMBER 2021

www.sosinclasses.com +91 90000 36699 [email protected]

Pag

e72

▪ Arsenicosis is the medical word for arsenic poisoning, which occurs due to accumulation of large amounts of arsenic in the body.

▪ It leads to adverse health effects through inhibition of essential enzymes, which ultimately leads to death from multi-system organ failure.

▪ Long-term exposure to arsenic from drinking-water and food can cause cancer and skin lesions. It has also been associated with cardiovascular disease and diabetes.

▪ In utero and early childhood exposure has been linked to negative impacts on cognitive development and increased deaths in young adults.

Steps Taken: Under the 2030 Agenda for Sustainable Development, the indicator of "safely managed drinking water services" calls for tracking the population accessing drinking water which is free of faecal contamination and priority chemical contaminants, including arsenic. Jal Jeevan Mission is envisioned to provide safe and adequate drinking water through individual household tap connections by 2024 to all households in rural India. Recently, the Jal Jeevan Mission (Urban) has also been launched.

Way Forward: There is an urgent need to monitor irrigation water quality along with drinking water. What is required is to make its mitigation a success by involving the public in planning and maintenance, while also giving states the necessary push. The remedial measures include a variety of options, ranging from removing arsenic from groundwater after it is extracted, searching alternative aquifers, reducing the level within the aquifer itself, dilution of the contaminants by artificial recharge, blending with potable water etc.

2.8. ANALYSE THE UNITED IN SCIENCE 2021 REPORT.

Context: Recently, the World Meteorological Organization (WMO) released a report named United in Science 2021. It is a multi-organization high-level compilation of the latest climate science information. The report is coordinated by the WMO, with inputs from the United Nations Environment Programme, World Health Organization, Intergovernmental Panel on Climate Change, Global Carbon Project, World Climate Research Programme and the Met Office (UK).

Climate Change:

▪ The pace of climate change has not been slowed by the global Covid-19 pandemic and the world remains behind in its battle to cut carbon emissions.

▪ It has caused only a temporary downturn in carbon dioxide (CO2) emissions in 2020. ▪ High latitude regions and the Sahel are likely to be wetter over 2021–2025, than the

recent past. ▪ Reduction targets are not being met and there is a rising likelihood the world will miss

its Paris Agreement target of reducing global warming to 1.5 degrees Celsius above pre-industrial levels.

▪ There is an increasing likelihood that temperatures would temporarily breach the threshold of 1.5 degrees Celsius above the pre-industrial era, in the next five years.

Page 76: MONTHLY NEWS DIARY SEPTEMBER 2021

MONTHLY NEWS DIARY SEPTEMBER 2021

www.sosinclasses.com +91 90000 36699 [email protected]

Pag

e73

Temperature:

➢ Average global temperature for the past five years was among the highest on record. ➢ Rising global temperatures are fuelling devastating extreme weather throughout the

world, with spiralling impacts on economies and societies. ➢ Climate hazards such as heatwaves, wildfires, and poor air quality combine to threaten

human health worldwide, putting vulnerable populations at particular risk.

Greenhouse Gases: Concentrations of major greenhouse gases in the atmosphere continued to increase last year and during the first half of 2021.

Fossil Fuel Emissions: Fossil fuel emissions from coal, gas, cement, etc were back to 2019 levels or even higher in 2021.

Sea Level: Global mean sea levels rose 20 cm from 1900 to 2018. Even if emissions are reduced to limit warming to well below 2°C, global mean sea level would likely rise by 0.3-0.6 m by 2100 and could rise 0.3-3.1 m by 2300.

Loss of Work Hours: An excess of 103 billion potential work hours were lost globally in 2019, compared to 2000. It was due to heat-related mortality and work impairment, caused by rising temperatures.

Suggestions:

➢ More countries should develop long-term strategies that are consistent with the 2015 Paris Agreement.

Page 77: MONTHLY NEWS DIARY SEPTEMBER 2021

MONTHLY NEWS DIARY SEPTEMBER 2021

www.sosinclasses.com +91 90000 36699 [email protected]

Pag

e74

➢ Net-zero commitments needed to be translated into strong near-term policies and action.

➢ Adaptation strategies are needed where they do not exist – especially in low-lying coasts, small islands, deltas and coastal cities.

➢ Covid-19 recovery efforts should be aligned with national climate change and air quality strategies to reduce risks from compounding and cascading climate hazards and gain health co-benefits.

Way Forward

World needs a breakthrough on protecting people and their livelihoods, with at least half of all public climate finance committed to building resilience and helping people adapt. And it needs much greater solidarity, including full delivery of the long-standing climate finance pledge to help developing countries take climate action.

2.9. Explain the significance of the Mura-Drava-Danube (MDD).

Background: Recently, Mura-Drava-Danube (MDD) was declared as the world’s first ‘five-country biosphere reserve’ by the United Nations Educational, Scientific and Cultural Organization (UNESCO).

Importance of the MDD:

The area is one of the richest in Europe in terms of species diversity. It is home to floodplain forests, gravel and sand banks, river islands, oxbows and meadows.The area is home to the highest density in Europe of breeding pairs of white-tailed eagle and endangered species such as the little tern, black stork, otters, beavers and sturgeons. It is also an important steppingstone for more than 2,50,000 migratory waterfowls every year.

About MDD: The biosphere reserve covers 700 kilometres of the Mura, Drava and Danube rivers

and stretches across Austria, Slovenia, Croatia, Hungary and Serbia. The total area of the reserve is a million hectares - in the so-called ‘Amazon of

Europe’, which is now the largest riverine protected area in Europe. The biosphere “represented an important contribution to the European Green Deal

(climate action plan) and contributed to the implementation of the EU Biodiversity Strategy in the Mura-Drava-Danube region.”

The strategy’s aim is to revitalise 25,000 km of rivers and protect 30% of the European Union’s land area by 2030.

Page 78: MONTHLY NEWS DIARY SEPTEMBER 2021

MONTHLY NEWS DIARY SEPTEMBER 2021

www.sosinclasses.com +91 90000 36699 [email protected]

Pag

e75

Biosphere Reserve (BR):

➢ BR is an international designation by (UNESCO) for representative parts of natural and cultural landscapes extending over large areas of terrestrial or coastal/marine ecosystems or a combination of both.

➢ BR tries to balance economic and social development and maintenance of associated cultural values along with the preservation of nature.

➢ BRs are nominated by national governments and remain under the sovereign jurisdiction of the states where they are located.

➢ These are designated under the intergovernmental MAB Programme by the Director-General of UNESCO following the decisions of the MAB International Coordinating Council (MAB ICC).

➢ The Man and the Biosphere (MAB) Programme is an intergovernmental scientific programme that aims to establish a scientific basis for enhancing the relationship between people and their environments. Their status is internationally recognized.

➢ There are 727 biosphere reserves in 131 countries, including 22 transboundary sites.

Three Main Zones:

1. Core Areas: It comprises a strictly protected zone that contributes to the conservation of landscapes, ecosystems, species, and genetic variation.

2. Buffer Zones: It surrounds or adjoins the core area(s) and is used for activities compatible with sound ecological practices that can reinforce scientific research, monitoring, training and education.

3. Transition Area: The transition area is where communities foster socio-culturally and ecologically sustainable economic and human activities.

Biosphere Reserves in India:

Presently, there are 18 Biosphere Reserves in India, among which 12 Biosphere reserves in India find their place in UNESCO’s List of Man & Biosphere Reserves Programme. The latest included under the MAB was ‘Panna Biosphere Reserve’ (Madhya Pradesh).

Page 79: MONTHLY NEWS DIARY SEPTEMBER 2021

MONTHLY NEWS DIARY SEPTEMBER 2021

www.sosinclasses.com +91 90000 36699 [email protected]

Pag

e76

Page 80: MONTHLY NEWS DIARY SEPTEMBER 2021

MONTHLY NEWS DIARY SEPTEMBER 2021

www.sosinclasses.com +91 90000 36699 [email protected]

Pag

e77

3. SCIENCE & TECHNOLOGY 3.1. UNDER THE LUNAR SCIENCE WORKSHOP 2021, WHAT ARE THE FINDINGS OF

CHANDRAYAAN-2 ORBITER?

Background: Recently, the Lunar Science Workshop 2021, was conducted by Indian Space Research Organisation (ISRO) to commemorate the completion of two years of operation of Chandrayaan-2 orbiter in lunar orbit. According to the ISRO, the observations of the Chandrayaan-2 orbiter payloads have yielded discovery-class findings. The Chandrayaan-3 mission is likely to be launched late next year.

About the Chandrayaan-2:

➢ Lunar Exploration: It is the second spacecraft in the Indian series of Lunar exploration satellites. It comprised an orbiter, lander named Vikram, and rover named Pragyan to explore the unexplored South Polar region of the Moon.

➢ Launch: It was launched on 22nd July 2019 from the Sriharikota spaceport by GSLV Mk-III. It was inserted into a lunar orbit in August 2019. The orbiter and lander modules were separated as two independent satellites in September 2019.

➢ Failure of Lander: Vikram lander's descent was as planned, and normal performance was observed up to an altitude of 2.1 km from Lunar surface in September 2019. Subsequently, communication from the lander was lost and the lander had a hard landing on the lunar surface. The six-wheeled Pragyan rover was accommodated inside the Vikram lander. A successful soft-landing would have made India the fourth country after the erstwhile Soviet Union, the United States, and China to do so.

➢ Role of Orbiter: The orbiter, placed in its intended orbit around the Moon, provided the understanding of the Moon's evolution and mapping of minerals and water molecules in polar regions, using its eight advanced scientific instruments. The precise launch and optimized mission management have ensured a long life of almost seven years for the orbiter instead of the planned one year.

Findings of Chandrayaan-2 Orbiter:

✓ Detection of Argon-40: The mass spectrometer Chandra’s Atmospheric Compositional Explorer 2 (CHACE 2) conducted first-ever in-situ study of the composition of the lunar neutral exosphere from a polar orbital platform. It detected and studied the variability of the Argon-40 at the middle and higher latitudes of the Moon, depicting the radiogenic activities in the mid and higher latitudes of the Lunar interior.

Note:

• The ‘PRADAN’ portal is hosted by Indian Space Science Data Centre (ISSDC), the nodal centre of data archive for ISRO missions.

• The data from the Chandrayaan-2 mission is being released for the wider public use through the PRADAN portal.

Page 81: MONTHLY NEWS DIARY SEPTEMBER 2021

MONTHLY NEWS DIARY SEPTEMBER 2021

www.sosinclasses.com +91 90000 36699 [email protected]

Pag

e78

✓ Detection of Chromium and Manganese: Chandrayaan-2 Large Area Soft X-ray Spectrometer (CLASS) payload has detected minor elements of chromium and manganese through remote sensing.

✓ Observations of Microflares of the Sun: The observations of microflares of the Sun, during the quiet-Sun period, which provide important clues on the coronal heating problem of the Sun, were made by the Solar X-ray Monitor (XSM) payload.

✓ Detection of the Hydration Features: The first-ever unambiguous detection of the hydration features of the Moon was achieved by Chandrayaan-2 with its Imaging Infra-Red Spectrometer (IIRS) payload, which captured clear signatures of Hydroxyl and water-ice on the lunar surface.

✓ Detection of the Subsurface Water-ice: The Dual Frequency Synthetic Aperture Radar (DFSAR) instrument detected signatures of the subsurface water-ice and achieved high resolution mapping of the lunar morphological features in the polar regions.

✓ Imaging the Moon: Imaging the moon from 100 km lunar orbit with "best-ever" achieved resolution of 25 cm with its Orbiter High Resolution Camera (OHRC).

✓ Geological Findings: The Terrain Mapping Camera (TMC 2) of Chandrayaan-2, which is conducting imaging of the Moon at a global scale, has found interesting geologic signatures of lunar crustal shortening, and identification of volcanic domes.

✓ Study of Moon's Ionosphere: The Dual Frequency Radio Science (DFRS) experiment onboard Chandrayaan-2 has studied the ionosphere of the Moon, which is generated by the solar photo-ionisation of the neutral species of the lunar exosphere.

3.2. RESTRAINING MOSQUITO POPULATIONS WITH CRISPR. EXPLAIN.

Context: Recently, researchers have created a system that restrains populations of mosquitoes by leveraging advancements in Clustered Regularly Interspaced Short Palindromic Repeats (CRISPR)-based genetic engineering. Mosquitoes infect millions each year with debilitating diseases such as dengue and malaria.

Sterile Insect Technique: SIT is an environmentally safe and proven technology to suppress wild populations. To further advance its utility, a novel CRISPR-based technology, termed Precision-guided Sterile Insect Technique (PgSIT) is described.

Page 82: MONTHLY NEWS DIARY SEPTEMBER 2021

MONTHLY NEWS DIARY SEPTEMBER 2021

www.sosinclasses.com +91 90000 36699 [email protected]

Pag

e79

PgSIT:

➢ It is a new scalable genetic control system that uses a CRISPR-based approach to engineer deployable mosquitoes that can suppress populations.

➢ Males don't transmit diseases, so the idea is to release more and more sterile males. ➢ The population of mosquitos can be suppressed without relying on harmful chemicals

and insecticides. ➢ It alters genes linked to male fertility—creating sterile offspring—and female flight in

Aedes aegypti, the mosquito species responsible for spreading diseases including dengue fever, chikungunya and Zika.

➢ PgSIT mechanistically relies on a dominant genetic technology that enables simultaneous sexing and sterilization, facilitating the release of eggs into the environment ensuring only sterile adult males emerge.

➢ The system is self-limiting and is not predicted to persist or spread in the environment, two safety features that should enable acceptance for this technology.

➢ PgSIT eggs can be shipped to a location threatened by mosquito-borne disease or developed at an on-site facility that could produce the eggs for nearby deployment.

➢ Once the PgSIT eggs are released in the wild, sterile PgSIT males will emerge and eventually mate with females, driving down the wild population as needed.

CRISPR:

▪ It is a gene editing technology, which replicates natural defence mechanisms in bacteria to fight virus attacks, using a special protein called Cas9.

▪ CRISPR-Cas9 technology behaves like a cut-and-paste mechanism on DNA strands that contain genetic information. The specific location of the genetic codes that need to be changed, or edited, is identified on the DNA strand, and then, using the Cas9 protein, which acts like a pair of scissors, that location is cut off from the strand.

▪ A DNA strand, when broken, has a natural tendency to repair itself. Scientists intervene during this auto-repair process, supplying the desired sequence of genetic codes that binds itself with the broken DNA strand.

▪ CRISPR-Cas9 is a simple, effective, and incredibly precise technology with potential to revolutionize human existence in future.

▪ Emmanuelle Charpentier of France and Jennifer A Doudna of the USA were awarded the 2020 Nobel Prize in Chemistry for developing CRISPR/Cas9 genetic scissors.

Page 83: MONTHLY NEWS DIARY SEPTEMBER 2021

MONTHLY NEWS DIARY SEPTEMBER 2021

www.sosinclasses.com +91 90000 36699 [email protected]

Pag

e80

3.3. CRITICALLY ANALYSE THE ROLE OF ROBOTS IN WARFARE.

Background: Recently, Israel Aerospace Industries unveiled a remote-controlled armed robot ‘REX MKII’, which can patrol battle zones, track infiltrators and open fire. The use of robots in the war involves dealing with ethical dilemmas. The proponents say that such semi-autonomous machines allow armies to protect their soldiers, while critics fear this marks another dangerous step toward robots making life-or-death decisions.

Arguments in Favour of Use of Robots in War:

▪ No Physiological Limitations: Autonomous robots, because they are not physiologically limited, can operate without sleep or food, perceive things that people do not, and move in ways that humans cannot. The use of a broad range of robotic sensors is better equipped for battlefield observations than human sensory abilities.

▪ Operational Benefits to the Military: The robots provide following benefits: faster, cheaper, better mission accomplishment; longer range, greater persistence, longer endurance, higher precision; faster target engagement; and immunity to chemical and biological weapons.

▪ Ability to Act Conservatively: Robots do not need to protect themselves in cases of low certainty of target identification. Autonomous armed robotic vehicles do not need to have self-preservation as a foremost drive, if at all. They can be used in a self-sacrificing manner if needed and appropriate, without reservation by a commanding officer.

▪ Minimising Loss of Human Life: Reducing loss of human lives forms one of the core principles of ethics of war, which can be accomplished by the use of the robots.

Arguments Against the Use of Robots in War:

➢ Lowering Threshold of Entry into War: The use of robot soldiers will cheapen the cost of war, making future wars more likely.

➢ The threshold of entry into warfare may be lowered as we will now be risking machines and fewer human soldiers. This could violate the conditions of just warfare.

➢ Errors in Targeting: Such weapons are worrisome because they can’t be trusted to distinguish between combatants and civilians or make proper calls about the harm attacks may do to nearby civilians.

About the REX MKII:

The robot can gather intelligence for ground troops, carry injured soldiers and supplies in and out of battle, and strike nearby targets.

The Israeli military is currently using a smaller but similar vehicle called the Jaguar to patrol the border with the Gaza Strip.

Unmanned ground vehicles are being increasingly used by other armies, including those of the United States, Britain and Russia.

Their tasks include logistical support, the removal of mines and firing weapons. An alternative Smart Wall has been proposed to replace the physical and armed

patrolling with advanced surveillance technology at the USA-Mexico border.

Page 84: MONTHLY NEWS DIARY SEPTEMBER 2021

MONTHLY NEWS DIARY SEPTEMBER 2021

www.sosinclasses.com +91 90000 36699 [email protected]

Pag

e81

➢ Ignoring Conventions of War: Machines cannot understand the value of human life, which in essence undermines human dignity and violates human rights laws. Therefore, machines are likely to commit atrocities and violate the basic rules of war like the Hague Conventions, and other declarations delimiting how a war should be fought.

➢ Persistent Risks: There will always be risks like proliferation of the technology to other nations and terrorists. Also, the robotic machines are prone to cyber-security attacks or hacking and they can be used against their own people.

Security Management in India:

➢ CIBMS Project: The Indian government has been pushing for technological solutions through the Comprehensive Integrated Border Management System (CIBMS) project. The purpose is to integrate technology with the existing systems to facilitate better detection and interception by the man behind the machine.

➢ National Counter Rogue Drones Guidelines 2019: To deal with possible security challenges from rogue drones to key installations like nuclear power plants and military bases.

Way Forward:

Technological revolution propelled by Artificial Intelligence, Machine learning, etc, is the need of the hour to enhance efficiency, productivity and optimization across industries and sectors. However, before deployment of robotics into warfare, thorough research needs to be done, so that opportunities can be maximized while keeping the humanitarian loss at the minimum.

3.4. EXPLAIN ABOUT THE MU VARIANT OF COVID-19.

Background: Recently, the World Health Organisation (WHO) has added a new variant of Covid-19 in the list of Variants of Interest (VOI) and named it Mu (B.1.621). It has also added C.1.2 as a new VOI. According to INSACOG (Indian SARS-CoV-2 Genomics Consortium), India has so far not seen Mu and C.1.2, and the Delta variant and its sub-lineages continue to be the main Variants of Concern (VOC). C.1.2 is a sub-lineage of the C.1 variant described in South Africa but did not spread globally.

About:

▪ Mu belongs to the lineage B.1.621 variant and named after the twelfth letter of the

Greek alphabet. It was first detected in Colombia in January 2021.

▪ It has a constellation of mutations that indicate potential properties of immune escape.

It has several substitutions affecting the Spike Protein and amino acid changes.

▪ It has seen mutations, E484K, N501Y, P681H, D614G, which have been reported in other

VOIs and VOCs. It is the fifth ‘VOI’ to be monitored by WHO. The other four VOIs are: Eta

(lineage B.1.525), Iota (lineage B.1.526), Kappa (lineage B.1.617.1), and Lambda (lineage

C.37).

Page 85: MONTHLY NEWS DIARY SEPTEMBER 2021

MONTHLY NEWS DIARY SEPTEMBER 2021

www.sosinclasses.com +91 90000 36699 [email protected]

Pag

e82

Variants of Interest:

A variant is placed in the VOI list if it is seen to have certain “genetic changes that are predicted or known to affect virus characteristics such as transmissibility, disease severity, immune escape, diagnostic or therapeutic escape”. To be added to the VOI list, a variant must also be “identified to cause significant community transmission or multiple Covid-19 clusters in multiple countries” and suggest “an emerging risk to global public health”.

Variants of Concern:

A VOI can become a VOC if it is demonstrated to be associated with an increase in transmissibility or virulence, or with a “decrease in effectiveness of public health and social measures or available diagnostics, vaccines, and therapeutics”. Currently, four variants of the coronavirus are designated as variants of concern: Alpha (lineage B.1.1.7, the so-called ‘UK variant’), Beta (lineage B.1.351, ‘South Africa variant’), Gamma (lineage P.1, ‘Brazil variant’), Delta (lineage B.1.617.2).

3.5. EXAMINE THE ROLE OF RICE IN BOOSTING NOURISHMENT.

Background: According to a recent study, as many as 12 folk varieties of Indian rice examined by researchers can supplement the nutritional demand of important Fatty Acids (FA) in undernourished mothers. Rice contains various classes of fatty acids, vitamins, minerals, starch and a small amount of protein.

Fatty Acids:

▪ Fatty acids are natural components of fats and oils. These have many important functions in the body, including energy storage.

▪ Based on their chemical structure they can be differentiated into three groups: ‘saturated’, ‘mono-unsaturated’ and ‘poly-unsaturated’ fatty acids.

Mutation, Variant and Strain When a virus replicates, it doesn’t always manage to produce an exact copy of itself. This means that, over time, the virus may start to differ slightly in terms of its genetic sequence. Any changes to the viral genetic sequence during this process is known as a Mutation. Viruses with new mutations are sometimes called Variants. Variants can differ by one or multiple mutations. When a new variant has different functional properties to the original virus and becomes established in a population, it is sometimes referred to as a New Strain of the virus. All strains are variants, but not all variants are strains.

Page 86: MONTHLY NEWS DIARY SEPTEMBER 2021

MONTHLY NEWS DIARY SEPTEMBER 2021

www.sosinclasses.com +91 90000 36699 [email protected]

Pag

e83

▪ Saturated fatty acids (fats) are mainly found in animal foods, such as (fatty) meat, lard, sausage, butter and cheese but even in palm kernel and coconut oil, which are used for frying.

▪ Most unsaturated fatty acids (fats) are of plant and fatty fish origin. Meat products contain both saturated and unsaturated fats.

▪ Within the family of PolyUnsaturated Fatty Acids (PUFAs), there are two different groups: the ‘omega-3-fatty acids ‘and ‘omega-6-fatty acids’.

▪ Both are considered essential fatty acids because they cannot be synthesized by humans.

▪ Trans fatty acids, more commonly called trans fats, are made by heating liquid vegetable oils in the presence of hydrogen gas and a catalyst, a process called hydrogenation. These are the worst types of fat for the heart, blood vessels, and rest of the body.

Findings of the Study:

➢ Helpful for Health: The traditional rice varieties can add essential FAs in the staple diet helping in the normal brain development in infants. Many folk varieties like Athikaraya, Dudh-sar, Kayame, Neelam samba, Srihati, Maharaji and Bhejri are known in folk medicine to enhance milk production in lactating women. Varieties like Kelas, DudheBolta and Bhutmoori are rich in iron and can be included in the diet of mothers to treat anaemia.

➢ Address the Problem of Undernutrition: The traditional varieties help address the problem of undernutrition in under five children. India was placed at the 94th spot among 107 countries by the Global Hunger Index 2020. It is calculated on the basis of total undernourishment of the population, child stunting, wasting and child mortality.

➢ Contributes to the Economy: Recently, the first export consignment of Bao-dhaan (red rice) from Assam was sent to the US in March 2021. This enhances incomes of farming families. This iron-rich red rice is grown in the Brahmaputra valley of Assam, without the use of any chemical fertilizer.

➢ Resistance against Disease: Seven rice varieties of North East India — Meghalaya lakang, Chingphourel, Manuikhamei, Kemenyakepeyu, Wainem, Thekrulha, and Koyajang — have the potential to resist leaf and neck blast disease in rice plants. Blast disease caused by fungal pathogen Pyricularia oryzae is a major threat to rice productivity worldwide.

➢ Conservation is Cheaper: In situ conservation of these neglected and vanishing varieties of rice, rich in nutrients, is a cheaper option than High-Yielding Varieties (HYVs). HYV seeds were developed by scientists to improve food supplies and reduce famine in developing countries. In situ and ex situ conservation focuses on the maintenance of species diversity within or away from their natural habitats, respectively.

Page 87: MONTHLY NEWS DIARY SEPTEMBER 2021

MONTHLY NEWS DIARY SEPTEMBER 2021

www.sosinclasses.com +91 90000 36699 [email protected]

Pag

e84

4. PRELIMS WORKBOOK 1. Which one among the following statements best describes the Hycean planet? a) It is a dwarf planet recently discovered by NASA’s Transiting Exoplanet Survey Satellite

(TESS). b) It is a minor planet between the orbits of Mars and Jupiter. c) It is a habitable planet described as a hot, water-covered planet with a hydrogen-rich

atmosphere. d) It is a new exotic planet outside our solar system in the constellation Cassiopeia. 2. Consider the following statements with respect to Komodo dragon: 1) It is the largest living lizard species in the world. 2) It is endemic to the Amazon Forest. 3) Komodo National Park was classified as a UNESCO World Heritage Site in 1991.

Which of the above statements is/are incorrect?

a) 1 and 2 only b) 2 only c) 2 and 3 only d) 3 only 3. Which of the following is/are the applications of Doppler radar? 1) Radiology and healthcare 2) Weather Forecasting 3) Submarines 4) Aviation

Options:

a) 1, 2 and 3 only b) 1, 2 and 4 only c) 2, 3 and 4 only d) 1, 2, 3 and 4 4. Which is the best description of NAMASYA Mobile App?

a) It is an app developed exclusively for the benefit of NALCO’s MSE Vendors. b) It is an electronic marketplace providing a platform to connect aqua farmers and the

buyers. c) It is an app to log in and track the grievances of the consular services offered by the Indian

embassies in foreign locations. d) It is an app where citizens can pay homage and contribute financially to the bereaved

families of soldiers 5. With reference to Manda buffalo, which of the following statements is/are correct? 1) They are found in the Western Ghats. 2) The Manda are resistant to parasitic infections and are less prone to diseases.

Options:

a) 1 only b) 2 only c) Both d) None 6. With reference to Minimum Support Price (MSP), which of the following statements

is/are correct?

1) Minimum Support Price was introduced by the Centre for the first time in the year 1991 to

reduce the impact on farming because of LPG reforms.

2) The government announces the MSP at the start of each cropping season.

3) MSP doesn’t have any legal stature.

Options:

a) 1 only b) 2 and 3 only c) 1 and 3 only d) None

Page 88: MONTHLY NEWS DIARY SEPTEMBER 2021

MONTHLY NEWS DIARY SEPTEMBER 2021

www.sosinclasses.com +91 90000 36699 [email protected]

Pag

e85

7. Which of the following pollutants is/are emitted from cement industries?

1) Particulate Matter 2) Carbon Monoxide 3) Sulphur Dioxide 4) Nitrogen Dioxide

Options:

a) 1, 2 and 3 only b) 2, 3 and 4 only c) 1, 3 and 4 only d) 1, 2, 3 and 4 8. Which of the following chemicals are barred from usage in firecrackers?

1) Antimony 2) Lithium 3) Mercury 4) Arsenic 5) Lead Options:

a) 1, 3 and 4 only b) 2, 3, 4 and 5 only c) 1 and 5 only d) 1, 2, 3, 4 and 5 9. Recently the term ‘Akash Prime’ was seen in the news. What does it refer to?

a) A collaboration between Amazon Prime and the Education Ministry to run online classes b) A new version of the Akash surface-to-air missile developed by the DRDO c) A weather monitoring system deployed by India Meteorological Department d) An earth-observation satellite launched by ISRO for military applications 10. Which of the following are the reasons/factors for exposure to benzene pollution?

1) Automobile exhaust 2) Tobacco smoke 3) Wood burning 4) Using varnished wooden furniture 5) Using products made of polyurethane

Select the correct answer using the given code below: a) 1, 2 and 3 only b) 2 and 4 only c) 1, 3 and 4 only d) 1, 2, 3, 4 and 5 11. Consider the following statements with respect to COVAX: 1) It is a global science initiative established in 2008 that provides open-access to genomic

data of influenza viruses and the coronavirus responsible for the COVID-19 pandemic. 2) It is co-led by Gavi, the Coalition for Epidemic Preparedness Innovations (CEPI) and WHO. 3) COVAX is one of three pillars of the Access to COVID-19 Tools (ACT) Accelerator.

Which of the given statement/s is/are correct? a) 2 and 3 only b) 1 and 2 only c) 2 only d) 1, 2 and 3 12. Which of the following statements about Foot and Mouth Disease is/are correct? 1) Humans are not infected by the foot-and-mouth disease virus. 2) National Animal Disease Control Programme for FMD and Brucellosis (NADCP) is a Central

Sector Scheme where 100% of funds shall be provided by the Central Government to the States/UTs. Options:

a) 1 only b) 2 only c) Both d) None 13. Consider the following statements: 1) PM2.5 and PM10 are capable of penetrating deep into the lungs but PM2.5 can even enter

the bloodstream, primarily resulting in cardiovascular and respiratory impacts.

Page 89: MONTHLY NEWS DIARY SEPTEMBER 2021

MONTHLY NEWS DIARY SEPTEMBER 2021

www.sosinclasses.com +91 90000 36699 [email protected]

Pag

e86

2) As of now National Ambient Air Quality Standards (NAAQS) do not meet the WHO’s existing standards. Which of the above statements is/are correct?

a) 1 only b) 2 only c) Both d) None 14. Fast and Secured Transmission of Electronic Records (FASTER) recently seen in news is: a) An electronic toll collection system operated by the National Highways Authority of India b) A system conceived by Supreme Court to ensure bail orders reach jail authorities swiftly c) An Investor Facilitation Portal for Ease of Doing Business d) A secure cloud based platform for storage, sharing and verification of documents 15. With respect to Arjun Mk-1A battle tanks, which of the following statements is/are

correct? 1) It is a second-generation main battle tank manufactured in India in collaboration with

Russia. 2) India has recently agreed to export the battle tanks to Vietnam to boost India’s material

exports. Options:

a) 1 only b) 2 only c) Both d) None 16. Which of the following committees are associated with preserving the ecology and

conservation of Western Ghats? 1) Dr. K. Kasturirangan Committee 2) Pitamber Committee 3) Madhav Gadgil Committee 4) BP Jeevan Reddy Committee

Options: a) 1, 2 and 3 only b) 1 and 3 only c) 2, 3 and 4 only d) 3 only 17. With reference to Asian Development Bank (ADB), which of the following statements

is/are incorrect? 1) India is a founding member of the Asian Development Bank. 2) Japan holds the largest share in ADB, followed by China and India. 3) Only countries from Asia can be part of ADB.

Options: a) 1 only b) 2 and 3 only c) 3 only d) 1, 2 and 3 18. Which of the following is/are the Nerve agents? 1) Sarin (GB) 2) Soman (GD) 3) Tabun (GA) 4) VX

Options: a) 1 only b) 2 and 4 only c) 1, 2 and 3 only d) 1, 2, 3 and 4 19. In the context of modern scientific research, consider the following statements about

'IceCube', a particle detector located at South Pole, which was recently in the news: 1) It is the world’s largest neutrino detector, encompassing a cubic kilometer of ice. 2) It is a powerful telescope to search for dark matter. 3 It is buried deep in the ice.

Which of the statements given above is/are correct? a) 1 only b) 2 and 3 only c) 1 and 3 only d) 1, 2 and 3 20. With respect to Global Innovation Index, which of the following statements is/are

correct?

Page 90: MONTHLY NEWS DIARY SEPTEMBER 2021

MONTHLY NEWS DIARY SEPTEMBER 2021

www.sosinclasses.com +91 90000 36699 [email protected]

Pag

e87

1) It is published by World Economic Forum. 2) GII was published for the first time in 2007. The Index is published biennially ever since. 3) India has been on a rising trajectory, over the past several years in the Global Innovation

Index (GII). Options:

a) 1 and 2 only b) 2 and 3 only c) 3 only d) None

ANSWER KEY

GS – 1

1) D

2) A

3) B

4) D

5) C

6) B

7) C

8) C

9) D

10) C

GS – 2

1) C

2) D

3) C

4) D

5) C

6) C

7) A

8) C

9) D

10) B

11) A

12) A

13) C

14) D

15) D

16) B

17) B

18) A

19) D

20) A

GS – 3

1) C

2) B

3) B

4) A

5) B

6) B

7) D

8) D

9) B

10) D

11) A

12) B

13) C

14) B

15) D

16) B

17) B

18) D

19) D

20) C

Page 91: MONTHLY NEWS DIARY SEPTEMBER 2021

MONTHLY NEWS DIARY SEPTEMBER 2021

www.sosinclasses.com +91 90000 36699 [email protected]

Pag

e88

Page 92: MONTHLY NEWS DIARY SEPTEMBER 2021

MONTHLY NEWS DIARY SEPTEMBER 2021

www.sosinclasses.com +91 90000 36699 [email protected]

Pag

e89